Формулы релятивистской механики: Основные формулы релятивистской механики

Содержание

Основные формулы релятивистской механики

Определение 1

Релятивистская механика – это раздел механики, в который превращаются законы Ньютона в случае, если материальное тело движется со скоростью, максимально близкой к скорости света.

Рисунок 1. Релятивистская механика материальной точки. Автор24 — интернет-биржа студенческих работ

На таких сверхвысоких скоростях с физическими вещами начинают происходить совершенно неожиданные и волшебные процессы, такие как замедления времени и релятивистское сокращение длины.

В пределах исследования релятивистской механики меняются формулировки некоторых устоявшихся в физике физических величин.

Данная формула, которую знает практически каждый человек, показывает, что масса является абсолютной мерой энергии тела, а также демонстрирует принципиальную вероятность перехода энергетического потенциала вещества в энергию излучения.

Основной закон релятивистской механики в виде материальной точки записывается так же, как и второй закон Ньютона: $F=\frac{dp}{dT}$.

Принцип относительности в релятивистской механике

Рисунок 2. Постулаты теории относительности Эйнштейна. Автор24 — интернет-биржа студенческих работ

Принцип относительности Эйнштейна подразумевает инвариантность всех существующих законов природы по отношению к постепенному переходу от одной инерциальной концепции отсчета к другой. Это означает, что все описывающие природные законы формулы должны быть полностью инвариантны относительно преобразований Лоренца. К моменту возникновения СТО теория, удовлетворяющая данному условию, уже была представлена классическая электродинамика Максвелла. Однако все уравнения ньютоновской механики оказались абсолютно неинвариантными относительно других научных постулатов, и поэтому СТО требовала пересмотра и уточнения механических закономерностей.

Готовые работы на аналогичную тему

В основу такого важного пересмотра Эйнштейн озвучил требования выполнимости закона сохранения импульса и внутренней энергии, которые находятся в замкнутых системах. Для того, чтобы принципы нового учения выполнялся во всех инерциальных концепциях отсчета, оказалось важным и первостепенным изменить определение самого импульса физического тела.

Если принять и использовать такое определение, то закон сохранения конечного импульса взаимодействующих активных частиц (например, при внезапных соударениях) начнет выполняться во всех инерциальных системах, непосредственно связанных преобразованиями Лоренца. При $β → 0$ релятивистский внутренний импульс автоматически переходит в классический. Масса $m$, входящая в основное выражение для импульса, является фундаментальная характеристика мельчайшей частицы, не зависящая от дальнейшего выбора концепции отсчета, а, следовательно, и от коэффициента ее движения.

Релятивистский импульс

Рисунок 3. Релятивистский импульс. Автор24 — интернет-биржа студенческих работ

Релятивистский импульс не пропорционален начальной скорости частицы, а его изменения не зависят от возможного ускорения взаимодействующих в инерциальной системе отчета элементов.2})\frac{3}{2}$

Если скорость определенной классической частицы беспредельно увеличивается под действием стабильной силы, то скорость релятивистского вещества не может в итог превысить скорость света в абсолютной пустоте. В релятивистской механике, так же, как и в законах Ньютона, выполняется и реализуется закон сохранения энергии. Кинетическая энергия материального тела $Ek$ определяется через внешнюю работу силы, необходимую для сообщения в будущем заданной скорости. Чтобы разогнать элементарную частицу массы m из состояния покоя до скорости под влиянием постоянного параметра $F$, эта сила обязана совершить работу.

Чрезвычайно важный и полезный вывод релятивистской механики состоит в том, что находящаяся в постоянном покое масса $m$ содержит невероятный запас энергии. Это утверждение имеет различные практические применения, включая сферу ядерной энергии. Если масса любой частицы или системы элементов уменьшилась в несколько раз, то при этом должна выделиться энергия, равная $\Delta E = \Delta m • c^2.2}$.

Скорость света является всегда граничным значением. Быстрее света в принципе не может двигаться ни одно физическое тело.

Многие задачи и проблемы смогло бы решить человечество, если бы ученым удалось разработать универсальные аппараты, способные передвигаться со скоростью, приближающейся к скорости света. Пока же люди могут о таком чуде только мечтать. Но когда-нибудь полёт в космос или на другие планеты с релятивистской скоростью станет не вымыслом, а реальностью.

Релятивистская механика | Фізика – легко!

Специальная теория относительности (СТО) рассматривает взаимосвязь физических процессов только в инерциальных системах отсчёта (СО), то есть в СО, которые движутся относительно друг друга равномерно прямолинейно.

Общая теория относительности (ОТО) рассматривает взаимосвязь физических процессов в неинерциальных СО, то есть в СО, которые ускоренно движутся относительно друг друга.

Пространство
характеризует взаимное расположение тел;
пространство однородно, имеет три измерения;
все направления в пространстве равноправны.

Время
характеризует последовательность событий;
время имеет одно измерение;
время однородно и изотропно.

Постулаты теории относительности:

1.  Во всех инерциальных СО все физические явления происходят одинаково.

Т.е. все инерциальные СО равноправны. Никакие опыты в любой области физики не позволяют выделить абсолютную инерциальную СО.

2. Скорость света в вакууме одинакова во всех инерциальных СО и
не зависит от скорости источника света и наблюдателя (т.е. скорость света в вакууме инвариантна).

Скорость распространения света в вакууме является максимально возможной скоростью распространения или передачи любого взаимодействия:
с = 299792,5 км/с.

Относительность одновременности

Событие – это любое явление, происходящее в данной точке пространства в некоторый момент времени.
Задать событие означает задать точку в четырёхмерном пространстве «координаты – время», т.е. когда и где событие происходит.

В классической механике Ньютона время одинаково в любой инерциальной СО, то есть имеет абсолютное значение и не зависит от выбора СО.

В релятивистской механике время зависит от выбора СО.

События, происходящие одновременно в одной СО, могут не быть одновременными в другой СО, движущейся относительно первой.

Относительно двух часов, один из которых расположен на носу, а другой на корме корабля, событие (вспышка) происходит не одновременно. Часы А и Б синхронизированы и находятся на одинаковом расстоянии от источника света, расположенного между ними. Свет распространяется с одинаковой скоростью во всех направлениях, но часы фиксируют вспышку в разные моменты времени.

Пусть один наблюдатель находится внутри корабля (внутренний наблюдатель) в системе отсчёта К’, а второй вне корабля (внешний наблюдатель) в системе отсчёта К.
Система отсчёта К’ связана с кораблём и движется со скоростью v относительно неподвижной системы отсчёта К, которая связана с внешнем наблюдателем.

Если посередине корабля, который движется с некоторой скоростью v относительно внешнего наблюдателя, вспыхнет источник света, то для внутреннего наблюдателя свет достигает кормы и носа корабля одновременно. Т.е. в системе отсчёта К’ эти два события происходят одновременно.

Для внешнего наблюдателя корма будет “приближаться” к источнику света, а нос корабля – удаляться, и свет достигнет кормы раньше, чем носа корабля. Т.е. в системе отсчёта К эти два события происходят не одновременно.

Релятивистский закон сложения скоростей

Классический закон сложения скоростей в релятивистской механике применять нельзя (это противоречит второму постулату СТО), поэтому в СТО применяют релятивистский закон сложения скоростей.

Очевидно, что при скоростях, которые много меньше скорости света, релятивистский закон сложения скоростей принимает вид классического закона сложения скоростей.

Следствия постулатов теории относительности

1. Промежутки времени увеличиваются, время замедляется.

Замедление времени экспериментально показано при радиоактивном распаде ядер: радиоактивный распад ускоренных ядер замедлен по сравнению с радиоактивным распадом таких  же покоящихся ядер.

2. Размеры тел уменьшаются в направлении движения.

Из формулы видно, что самую большую длину тело имеет в неподвижной СО. Изменение длины тела во время движения называется лоренцово сокращение длины.

Как связаны масса и энергия

В литературе знаменитую формулу Эйнштейна пишут в 4-х вариантах, что свидетельствует о не очень её глубоком понимании.

Оригинальная формула появилась в небольшой заметке Эйнштейна в 1905 году:

 Эта формула имеет глубокий физический смысл. Она говорит о том, что масса тела, которое находится в состоянии покоя как целое, определяет содержание энергии в нём, независимо от природы этой энергии.

Например, внутренняя кинетическая энергия хаотического движения частиц, из которых состоит тело, входит в энергию покоя тела, в отличие от кинетической энергии поступательного движения. То есть, нагревая тело, мы увеличиваем его массу.

Также следует обратить внимание на то, что формула читается справа налево любая масса определяет энергию тела. Но не всякая энергия может быть поставлена в соответствие с какой-нибудь массой.

Также из формулы следует, что

изменение энергии тела прямо пропорционально изменению его массы:

В случае, когда тело начинает двигаться, энергия покоя переходит в полную энергию в СО, которая движется поступательно как целое с определённой скоростью v.

Интересные статьи, в которых раскрыт смысл знаменитой формулы взаимосвязи массы и энергии можно прочесть сайте Германа Ароновича Розмана : “Существует ли “релятивистская масса”?” и “Может ли масса превращаться в энергию?”

Также стоит ознакомиться со статьей Л.Б Окуня “Понятие массы (Масса, энергия, относительность)”, опубликованной в журнале “Успехи физических наук”. Именно эта статья стала отправной точкой в “процессе наведения порядка в релятивистской механики”.

Так же рекомендую к просмотру фильм “Что такое теория относительности“.

Физические основы механики

Мы говорили, что скорость света — максимально возможная скорость распространения сигнала. Но что будет, если свет испускается движущимся источником в направлении его скорости V ? Согласно закону сложения скоростей, следующему из преобразований Галилея, скорость света должна быть равна c + V. Но в теории относительности это невозможно. Посмотрим, какой закон сложения скоростей следует из преобразований Лоренца. Для этого запишем их для бесконечно малых величин:

По определению скорости ее компоненты в системе отсчета K находятся как отношения соответствующих перемещений к временным интервалам:

Аналогично определяется скорость объекта в движущейся системе отсчета K’, только пространственные расстояния и временные интервалы надо взять относительно этой системы:

Следовательно, разделив выражение dx на выражение dt, получим:

Разделив числитель и знаменатель на dt’, находим связь

x-компонент скоростей в разных системах отсчета, которая отличается от галилеевского правила сложения скоростей:

Кроме того, в отличие от классической физики, меняются и компоненты скоростей, ортогональные направлению движения. Аналогичные вычисления для других компонент скоростей дают:

Таким образом, получены формулы для преобразования скоростей в релятивистской механике. Формулы обратного преобразования получаются при замене штрихованных величин на нештрихованные и обратно и заменой V на –V.

Теперь мы можем ответить на вопрос, поставленный в начале данного раздела. Пусть в точке 0′ движущейся системы отсчета

K’ установлен лазер, посылающий импульс света в положительном направлении оси 0’х’. Какой будет скорость импульса для неподвижного наблюдателя в системе отсчета К? В этом случае скорость светового импульса в системе отсчета К’ имеет компоненты

Применяя закон релятивистского сложения скоростей, находим для компонент скорости импульса относительно неподвижной системы К :

Мы получаем, что скорость светового импульса и в неподвижной системе отсчета, относительно которой источник света движется, равна

Тот же результат получится при любом направлении распространения импульса. Это естественно, так как независимость скорости света от движения источника и наблюдателя заложена в одном из постулатов теории относительности. Релятивистский закон сложения скоростей — следствие этого постулата.

Действительно, когда скорость движения подвижной системы отсчета << c, преобразования Лоренца переходят в преобразования Галилея, мы получаем обычный закон сложения скоростей

При этом ход течения времени и длина линейки будут одинаковы в обеих системах отсчета. Таким образом, законы классической механики применимы, если скорости объектов много меньше скорости света. Теория относительности не зачеркнула достижения классической физики, она установила рамки их справедливости.

Пример. Тело со скоростью v0 налетает перпендикулярно на стенку, двигающуюся ему навстречу со скоростью v. Пользуясь формулами для релятивистского сложения скоростей, найдем скорость v1 тела после отскока. Удар абсолютно упругий, масса стенки намного больше массы тела.

Воспользуемся формулами, выражающими релятивистский закон сложения скоростей.

Направим ось х вдоль начальной скорости тела v0 и свяжем систему отсчета K’ со стенкой. Тогда vx v0 и = –v. В системе отсчета, связанной со стенкой, начальная скорость v’0 тела равна

Поскольку стенку можно считать бесконечно массивной, по закону сохранения энергии после упругого удара тело отскочит в обратном направлении с тем же (относительно стенки) абсолютным значением скорости:

Вернемся теперь назад в лабораторную систему отсчета К. Подставляя в релятивистский закон сложения скоростей v’1 вместо v’x и учитывая опять же V = –v, находим после преобразований:

Проанализируем теперь предельные случаи.

  • Если скорости тела и стенки малы (v0 << с, v << с), то можно пренебречь всеми членами, где эти скорости и их произведение делятся на скорость света. Получаем тогда из найденной формулы результат классической механики

    Скорость шара после отскока увеличивается на удвоенную скорость стенки; направлена она, естественно, противоположно начальной. Ясно, что в релятивистском случае этот результат не годится. В частности, при v0 =  v = с/3 из него следует, что скорость тела после отскока будет равна v1 = –с, чего не может быть.

  • Пусть теперь на стенку налетает тело, двигающееся со скоростью света (например, лазерный луч отражается от двигающегося зеркала). Подставляя v0 = с в найденное соотношение, получаем

    Иными словами, скорость лазерного луча изменила направление, но не свою абсолютную величину, как и должно быть.

  • Рассмотрим теперь случай, когда стенка движется с релятивистской скоростью. В этом случае найденное соотношение дает нам

    Тело после отскока также будет двигаться со скоростью, близкой к скорости света.

  • Наконец, подставим в найденное соотношение значения v0 = v = с/3 :

    В отличие от классической механики, теория относительности дает для скорости после отскока значение, меньшее скорости света.

  • Напоследок посмотрим, что случится, если стенка удаляется от тела с той же скоростью (v = –v0). Имеем в этом случае:

    Как и в классической механике, тело стенку не догонит, и его скорость не изменится.

Время в классической и релятивистской физике. Основные формулы релятивистской механики

1 / 5

В классической механике пространственные координаты и время являются независимыми (при отсутствии голономных связей, зависящих от времени), время является абсолютным, то есть течёт одинаково во всех системах отсчёта, и действуют преобразования Галилея . В релятивистской же механике события происходят в четырёхмерном пространстве, объединяющем физическое трёхмерное пространство и время (пространство Минковского) и действуют преобразования Лоренца .{2}}{2}}} , нетрудно определить константу α {\displaystyle \alpha } .

В теории относительности выбор системы зависит от присутствия тел и их движения, которое необходимо описывать внутри выбранной системы отсчета. Вообще говоря, в современной физике и астрономии нет инерциальной системы отсчета. Можно лишь говорить о том насколько данная система близка к инерциальной.

Насколько отличается равномерный ход времени в различных системах отсчета связанных движущихся с обычными для современного человека скоростями? Можно ли заметить это? Пятьдесят лет назад ответы на эти вопросы были отрицательными. Часы, которые использовало человечество как в быту, так и в физических лабораториях для измерения времени представляли из себя примитивные механические приборы с погрешностью хода зачастую превышающей секунду в сутки. Их точность была слишком низкой чтобы заметить релятивистские эффекты в течении времени.

Есть два основных релятивистских эффекта, которые влияют на скорость хода времени. Первый – скорость. Если часы принадлежат разным системам отсчета, одна из которых движется относительно второй, то часы в первой системе будут идти медленнее. Если установить одновременность двух часов в некоторый момент времени, то так как темп течения времени в двигающейся системе будет медленнее, то часы в ней отстанут. Чеи длинее интервал времени между наблюдениями часов, тем значительнее отстают часы в движущейся системе отсчета. Скажем, для современного самолета, который летит со скоростью звука (300 м/сек) разность хода часов за один час полета составит наносекунды.

Второй эффект влияющий на скорость хода – разность гравитационных потенциалов. Двое покоющихся друг относительно друга часов, находящихся в разных точках пространства будут идти с разной скоростью. В том месте где сила гравитации слабее часы будут идти быстрее.

Пусть одни часы помещены на уровне моря, а вторые помещены на гору высотой 10 км. Тогда вторые часы будут идти быстрее и разность хода за час составит 3.6 наносекунды.

Регистрация хода часов с такой точностью стала возможна, когда были созданы атомные и водородные часы обладающие точностью хода не хуже чем на протяжении примерно одного часа.

Современные часы значительно точнее. С их помощью физикам удалось измерить неравномерность хода времени в двух различных точках пространства.

В одном случае это был эксперимент проведенный итальянскими учеными. Они синхронизовали двое часов. Одни часы они оставили на физическом факультете, а вторые на грузовике вывезли в горы и установили на высоте 3250 метрова над уровнем моря. Подождав 66 дней они спустили вторые часы и сравнили показания. Эксперимент показал полное согласие с теорией Эйнштейна! Часы, которые находились на горе ушли вперед, часы, которые остались на уровне моря – отстали.

Затем четверо идентичных часов были погружены на обычные самолеты и отправились в путешествие. Двое часов на восток, двое – на запад (поскольку полная скорость складывалсь из скорости самолета и скорости вращения Земли, то и скорости часов относительно инерциальной системы были различны). После облета земного шара часы выгрузили и сравнили их показания. Хотя ошибки измерения были достаточно велики (событие происходило в 1971 г.) сомнений быть не могло – эксперимент подтвердил предсказания теории относительности, подтвердил правоту А.Эйнштейна и установил экспериментальный базис для эффекта неравномерности хода часов.

В 1975 г. был поставлен специальный высокоточный эксперимент для измерения неравномерности хода часов на самолете, который летал над Чизапикском заливом (недалеко от устья реки Потомак, США). Точность хода часов к тому времени достигала . Самолет летал 15 часов, за это часы на борту опередили часы на Земле из – за эффектов неравномерности в изменяющемся гравитационном потенциале (самолет набирал высоту и снижался), а также неравномерность хода времени из – за движения системы отсчета относительно неподвижных часов. Часы, оставшиеся на Земле отсчитывали время находясь в гравитационном поле с большим значением потенциала, часы, находящиеся на борту самолета отсчитывали время в гравитационном поле с меньшим значением гравитационного потенциала. Эта разность хода часов за 15 часов полета достигла 53 наносекунд. В то же время часы, находящиеся на борту двигались относительно часов находящихся на поверхности Земли в покое, отставая от них. Этот эффект был значительно меньше. За 15 часов полета отставание составило всего 6 наносекунд. Оба эффекта в результате показали опережение хода часов в 47 наносекунд. Точность измерения неравномерности хода была лучше одного процента! Так в результате прямых измерений была продемонстрирована неоднородность хода времени в разных точках пространства и различных системах координат.

www.pereplet.ru/pops/sazhin/node3.html

Специальная теория относительности (СТО) рассматривает взаимосвязь физических процессов только в инерциальных системах отсчёта (СО), то есть в СО, которые движутся относительно друг друга равномерно прямолинейно.

Общая теория относительности (ОТО) рассматривает взаимосвязь физических процессов в неинерциальных СО, то есть в СО, которые ускоренно движутся относительно друг друга.

Пространство
характеризует взаимное расположение тел;
пространство однородно, имеет три измерения;
все направления в пространстве равноправны.

Время
характеризует последовательность событий;
время имеет одно измерение;
время однородно и изотропно.

Постулаты теории относительности:

1. Во всех инерциальных СО все физические явления происходят одинаково.

Т.е. все инерциальные СО равноправны . Никакие опыты в любой области физики не позволяют выделить абсолютную инерциальную СО.

2. Скорость света в вакууме одинакова во всех инерциальных СО и
не зависит от скорости источника света и наблюдателя (т.е. скорость света в вакууме инвариантна ).

Скорость распространения света в вакууме является максимально возможной скоростью распространения или передачи любого взаимодействия:
с = 299792,5 км/с.

Относительность одновременности

Событие – это любое явление, происходящее в данной точке пространства в некоторый момент времени.
Задать событие означает задать точку в четырёхмерном пространстве «координаты – время», т.е. когда и где событие происходит.

В классической механике Ньютона время одинаково в любой инерциальной СО, то есть имеет абсолютное значение и не зависит от выбора СО .

В релятивистской механике время зависит от выбора СО .

События, происходящие одновременно в одной СО, могут не быть одновременными в другой СО, движущейся относительно первой.

Относительно двух часов, один из которых расположен на носу, а другой на корме корабля, событие (вспышка) происходит не одновременно. Часы А и Б синхронизированы и находятся на одинаковом расстоянии от источника света, расположенного между ними. Свет распространяется с одинаковой скоростью во всех направлениях, но часы фиксируют вспышку в разные моменты времени.

Пусть один наблюдатель находится внутри корабля (внутренний наблюдатель) в системе отсчёта К’, а второй вне корабля (внешний наблюдатель) в системе отсчёта К.
Система отсчёта К’ связана с кораблём и движется со скоростью v относительно неподвижной системы отсчёта К , которая связана с внешнем наблюдателем .

Если посередине корабля, который движется с некоторой скоростью v относительно внешнего наблюдателя, вспыхнет источник света, то для внутреннего наблюдателя свет достигает кормы и носа корабля одновременно . Т.е. в системе отсчёта К’ эти два события происходят одновременно.

Для внешнего наблюдателя корма будет «приближаться» к источнику света, а нос корабля — удаляться, и свет достигнет кормы раньше, чем носа корабля . Т.е. в системе отсчёта К эти два события происходят не одновременно.

Релятивистский закон сложения скоростей

Классический закон сложения скоростей в релятивистской механике применять нельзя (это противоречит второму постулату СТО), поэтому в СТО применяют релятивистский закон сложения скоростей.

Очевидно, что при скоростях, которые много меньше скорости света, релятивистский закон сложения скоростей принимает вид классического закона сложения скоростей.

Следствия постулатов теории относительности

1. Промежутки времени увеличиваются, время замедляется.

Замедление времени экспериментально показано при радиоактивном распаде ядер: радиоактивный распад ускоренных ядер замедлен по сравнению с радиоактивным распадом таких же покоящихся ядер.

2. Размеры тел уменьшаются в направлении движения.

Из формулы видно, что самую большую длину тело имеет в неподвижной СО. Изменение длины тела во время движения называется лоренцово сокращение длины .

Как связаны масса и энергия

В литературе знаменитую формулу Эйнштейна пишут в 4-х вариантах, что свидетельствует о не очень её глубоком понимании.

Оригинальная формула появилась в небольшой заметке Эйнштейна в 1905 году:

Эта формула имеет глубокий физический смысл. Она говорит о том, что масса тела, которое находится в состоянии покоя как целое, определяет содержание энергии в нём, независимо от природы этой энергии.

Например , внутренняя кинетическая энергия хаотического движения частиц, из которых состоит тело, входит в энергию покоя тела, в отличие от кинетической энергии поступательного движения. То есть, нагревая тело, мы увеличиваем его массу.
Также следует обратить внимание на то, что формула читается справа налево любая масса определяет энергию тела. Но не всякая энергия может быть поставлена в соответствие с какой-нибудь массой.

Также из формулы следует, что

изменение энергии тела прямо пропорционально изменению его массы:

В случае, когда тело начинает двигаться, энергия покоя переходит в полную энергию в СО, которая движется поступательно как целое с определённой скоростью v .

Все законы классической механики справедливы для тел, движущихся со скоростями, которые намного меньше скорости света в вакууме. Если же скорость движения сравнима со скоростью света, то изучением такого движения занимается релятивистская механика.

В своей работе «Механика» Ньютон предполагал, что существует абсолютное пространство и абсолютное время. Неподвижная пустота, в которой находится Вселенная, и есть абсолютное пространство. Оно остаётся всегда одинаковым и неподвижным. А в нём равномерно течёт абсолютное время. Но великий учёный не указал, как обнаружить это абсолютное пространство и как доказать, что оно существует. Он считал, что доказательством может служить распространение света в пустоте. Ведь лучше всего он распространяется там, где ему не препятствует непрозрачное вещество. И пустое пространство идеально подходит для этого.

Но если это так, то скорость света в таком пространстве должна быть разной для наблюдателей, находящихся в разных точках. Ведь в таком пространстве для любого механического движения должны выполняться преобразования Галилея, согласно которым скорости движения изменяются при переходе от одной инерциальной системы отсчёта к другой. В классической механике скорость автомобиля по отношению к наблюдателю, стоящему на обочине дороги, отличается от его скорости по отношению к другому автомобилю, который движется в попутном или встречном направлении. Так, по отношению к встречному автомобилю его скорость будет равна сумме скоростей обоих автомобилей, а по отношению к попутному – разности их скоростей. По аналогии можно предположить, что и скорость света должна была бы быть разной для наблюдателей, движущихся в направлении его распространения и навстречу ему.

Но на самом деле всё совершенно не так. Неважно, в каком направлении распространяется свет. Независимо от положения наблюдателя его скорость всегда остаётся постоянной – 299 792 458 м/с (приблизительно 300 000 000 м/с). Это скорость света в вакууме. Она остаётся постоянной и относительно неподвижного перрона, и относительно, поезда, находящегося в движении.

Объяснить это явление классическая механика не могла. Это оказалось под силу лишь релятивистской механике Эйнштейна, более совершенной, чем механика Ньютона.

Новое учение Ньютона

На смену классической механике пришла специальная теория относительности – новое учение о пространстве и времени.

В классической механике пространство трёхмерно. Его называют евклидовым, а для его описания используют пространственные координаты x, y и z. Время же считается абсолютной, независимой от пространства величиной. И оно всегда идёт с одинаковой скоростью, где бы ни находились часы. Так считали до тех пор, пока в 1905 г. Альберт Эйнштейн не опубликовал свою статью «К электродинамике движущихся тел». В ней он изложил свою новую теорию, в которой доказал, что для наблюдателей, находящихся в движении, время движется медленнее, чем для тех, которые находятся в состоянии покоя. А если бы можно было достичь скорости света, то время остановилось бы. Это была совершенно новая теория, перевернувшая все представления в физике.

Преобразования Галилея оказываются верными только для тех объектов, скорость которых значительно ниже скорости света. Но если их скорость приближается к скорости света, то начинают проявляться релятивистские эффекты.

Релятивистская механика считает пространство четырёхмерным. Каждая точка этого пространства имеет 4 координаты: длину, ширину, высоту и время. Все они равноправны. Время в такой системе уже не является постоянной величиной. Скорость его течения зависит от скорости движения системы отсчёта.

В разных системах отсчёта, которые находятся в движении относительно друг друга, пространство и время выглядят по-разному. Для пересчёта координат пространства и времени из одной системы в другую используются преобразования Лоренца. В формулах пересчёта координаты пространства зависят от координат времени и наоборот. То есть, пространство и время неразделимы.

Релятивистские эффекты

Из преобразований Лоренца вытекают релятивистский эффект замедления времени и лоренцово сокращение длины.

Замедление времени

Этот удивительный эффект заключается в том, что при скоростях, сравнимых со скоростями света, время течёт с разной скоростью. И чем выше скорость объекта, тем медленнее течёт в нём время.

Количественное значение замедления времени получают из преобразований Лоренца:

где ∆t – время, проходящее между двумя событиями движущегося объекта, за которым следит неподвижный наблюдатель,

∆t o – время, проходящее между двумя событиями движущегося объекта с точки зрения наблюдателя, находящегося в движении,

v – относительная скорость движения объекта,

c – скорость света в вакууме.

Из формулы видно, что ∆ to ˃ ∆ t . То есть, для наблюдателя, находящегося в движении, время движется медленнее, чем для того, который находятся в состоянии покоя.

Очень наглядно эффект замедления времени проявляется в космических полётах, где движение происходит с релятивистскими скоростями. Ведь время на борту космического корабля течёт медленнее, чем на Земле. Так, если аппарат будет двигаться со скоростью, равной 0,95 скорости света, его полёт будет длиться 12 земных лет, но по часам на самом корабле пройдёт всего 7,3 года. А если корабль будет находиться в полёте 64 года по своему времени, то на Земле за это время пробежит уже 5 млн. лет. И кто знает, возможно, не только ход часов, но и ход всех процессов в полёте будет замедленным. И в будущем, возвратившись на Землю из длительного полёта, космонавты могут обнаружить, что их дети оказались старше их.

Лоренцово сокращение длины

Это сокращение называют также релятивистским сокращением длины движущегося тела или масштаба.

Длина любого объекта в релятивистской механике зависит от скорости. Этот эффект проявляется в том, что для наблюдателя предметы, движущиеся относительно него, имеют меньшую длину, чем в реальности. И чем больше скорость движения предмета, тем меньшим он кажется. При скорости, приближающейся к скорости света, длина предмета вдоль направления движения приближается к нулю. Именно поэтому наблюдатель, следящий за шаром, движущимся с такой скоростью, вместо него увидит плоский диск.

Следует уточнить, что эффект сокращения длины наблюдается только при скоростях, близких к скорости света.

Масса в релятивистской механике

В классической механике масса тела не зависит от скорости движения. А в релятивистской она растёт с увеличением скорости. Это видно из формулы:


где m o – масса тела в состоянии покоя;

m – масса тела в той инерциальной системе отсчёта, относительно которой оно движется со скоростью v;

с – скорость света в вакууме.

Отличие масс становится видным только при больших скоростях, приближающихся к скорости света.

Законы сохранения в релятивистской механике

Импульс тела

Импульс тела в релятивистской механике выглядит так:

В релятивистской механике выполняется закон сохранения релятивистского импульса. Этот импульс в замкнутой системе не изменяется с течением времени.

Взаимосвязь между массой и энергией

Эйнштейн установил связь между массой и энергией в релятивистской механике:

В состоянии покоя энергия систему равна:

E o = m o c 2

В специальной теории относительности выполняется закон сохранения релятивистской массы и энергии:

∆m = ∆E/c 2

Всякое изменение энергии тела или системы сопровождается изменением массы.

В классической механике масса является мерой инертности системы, а в релятивистской и мерой энергосодержания.

Кинетическая энергия

Кинетическая энергия при скоростях, приближающихся к скорости света, вычисляется как разность между кинетической энергией движущегося тела и кинетической энергией тела, находящегося в состоянии покоя:

где m – масса объекта;

v – скорость движения объекта;

c – скорость света в вакууме;

mc 2 – энергия покоя.

Данную формулу можно привести к такому виду:

При скоростях, значительно меньших скорости света, это выражение переходит в формулу кинетической энергии классической механики:

T = 1/2mv 2

Скорость света является предельным значением. Быстрее света не может двигаться ни одно тело.

Многие задачи смогло бы решить человечество, если бы удалось создать аппараты, способные передвигаться со скоростью, близкой к скорости света. Пока люди об этом только мечтают. Но когда-нибудь полёт с релятивистской скоростью станет реальностью.

Релятивистская ⚠️ механика: основной закон, постулаты, формулы

Мы понимаем принципы работы классической механики. Однако какие законы действуют на скоростях, близких к скорости света? В этой статье расскажем об основных законах релятивистской механики.

Что такое релятивистская механика

Релятивистская механика изучает движение частиц, у которых скорость близка к скорости света. Этот раздел появился на стыке механики, специальной теории относительности и общей теории относительности.

В основе лежат два постулата:

  1. Принцип общей относительности движения физических тел, под которым понимается равноправие всех инерциальных систем отсчета. Любая концепция отсчета, движущаяся относительно инерциальных принципов равномерно и прямолинейно, становится инерциальной системой отсчета. Движение со скоростью, постоянной относительно направления и модуля, называется прямолинейным. Так, связь этих методов указывает на то, что во всех законах физики эти системы будут одинаковы. Это утверждение получило название релятивистской инвариантности.
  2. Принцип стабильности скорости света в вакууме означает, что световая скорость в пустоте не зависит от движения самого источника света. Форсирование физических процессов является максимальной вероятностью для распространения материальных взаимодействий в дальнейшем.

Основной закон релятивистской механики

Сформулировать основную теорию релятивистской механики можно следующим образом: скорость изменения импульса физического тела равняется силе, действующей на точку. Такая формулировка этой теории формально совпадает со вторым законом Ньютона.

Первое положение универсальной теории относительности говорит о том, что формы фундаментальных постулатов физики во всех существующих инерциальных системах отсчета должны обязательно сохраняться.

В то же время основной закон динамики Ньютона выражается уравнением, в котором масса m является абсолютной и одинаковой во всех инерциальных системах отсчета. И при переходе от одной системы отсчета к другой форма записи закона также будет кардинально видоизменяться. Из чего можно сделать вывод о том, что этот закон не может стать базой для релятивистской динамики.

Эйнштейном было доказано, что форма второго закона Ньютона сохранится, если под общей массой понимать коэффициент, измеряющийся только в инерциальной системе отсчета при помощи материальной точки и скорости света в пустоте.2}\). То есть всякие изменения любой энергии (тела, системы тел, частицы) на \(ΔЕ\) сопровождается пропорциональным изменением массы на \(ΔЕ\).

При этом нельзя говорить, что масса переходит в энергию. Осуществляется переход из одной формы энергии в другую. Однако любое превращение энергии сопровождается превращением массы.

Источник: pixabay.com

Преобразование Лоренца

В классической физике при переходе от одной инерциальной системы отсчета к другой использовались преобразования Галилея. При этом очевидно, что они несовместимы со втором постулатом Эйнштейна, из которого следует, что скорость света во всех инерциальных системах одна и та же. В то же время следствием преобразований Галилея является классический закон сложения скоростей, которые не коррелирует с этим утверждением.
В классических преобразованиях \(t=t’\), то есть события, которые одновременны в одной системе отсчета, будут таковыми же и в любой другой. Но так ли это, если принять второй постулат Эйнштейна? Нет. Потому появилась необходимость в новых преобразованиях координат и времени, которые бы позволили переходить от одной системы отсчета к другой.

Преобразования, в основе которых лежат постулаты Эйнштейна, называются преобразованиями Лоренца.

С учетом того, что все инерциальные системы отсчета равноправны, преобразования Лоренца должны быть линейными относительно \(x, y, z, t \) и \( x’,y’, z’, t’\). Любая другая зависимость между этими элементами говорила бы о неравноправии систем отсчета. Линейный характер преобразований Галилея и Лоренца приводит к решению о том, что они должны различаться только коэффициентами пропорциональности. В преобразованиях Галилея этот коэффициент равен единице

\(x’ = x – Vt,\)

\(x = x’ + Vt’.\)

В свою очередь в преобразованиях Лоренца он равен \(y.\)

\(x’ = y(x – Vt),\)

\(x = y(x’ + Vt’).\)

Расчет для \(t\) в преобразованиях Лоренца будет иметь следующий вид:

\(t=\frac{t’+\frac{Vx’}{c^2}}{\sqrt[]{1-\frac{V^2}{c^2}}} .2}}.\)

Проекция скорости материальной точки на координатные оси в системе \(K\):

\(ux=\frac{dx}{dt}\)

\(uy=\frac{dy}{dt}\)

\(uz=\frac{dz}{dt}\)

Проекция скорости материальной точки на координатные оси в системе \(K\)’:

\(ux’=\frac{dx’}{dt}\)

\(uy’=\frac{dy’}{dt}\)

\(uz’=\frac{dz’}{dt}.\)

Принцип относительности в релятивистской механике

Принцип относительности – базовый постулат теории Эйнштейна, в котором говорится о том, что все физические процессы проходят единовременно и одинаково во всех инерциальных системах отсчета. Но в релятивистской механике теория относительности основывается не только на первом постулате, но и на втором, суть которого заключается в том, что скорость света в пустоте равна для всех существующих в той же среде инерциальных систем отсчета. На нее не влияют ни скорость светового сигнала, ни скорость самого источника.

Следствия, которые вытекают из постулатов теории относительности:

  • относительность расстояний между действующими объектами;
  • относительность промежутков времени;
  • замедление времени в движущихся системах отсчета.

Если вам нужна курсовая, дипломная работа по этой или другой теме, авторы ФениксХелп отлично справятся с такой задачей.

Релятивистская механика – Справочник химика 21

    Энергии hv фотона отвечает определенный импульс р. Поскольку фотон движется со скоростью света, то для Е и р должны применяться формулы релятивистской механики  [c.235]

    Движение частиц со скоростями, близкими к скорости света, рассматривают с позиции релятивистской механики, на основе теории относительности и законов [c.200]

    Сравнивая (21,2) с уравнением (21,3), мы видим, что переход от квантового уравнения к классическому соответствует формальному переходу к пределу й— 0, подобно переходу от релятивистской механики к нерелятивистской при с— оо. Поскольку й — величина постоянная, то такой предельный переход следует понимать условно. Он оправдывается только тогда, когда в уравнении (21,2) члены, содержащие й, малы по сравнению с остальными членами уравнения. [c.92]


    Одной из задач, которые ставит перед собой теоретическая физика, является вывод формул, описывающих то или иное явление или свойство (мы уже об этом упоминали). Итогом теории служит формула, связывающая физические величины разной природы. Физические величины имеют размерность. Уже из этого ясно, что формулы не могут содержать только безразмерные числа. Среди входящих в них размерных величин, по-видимому, должны присутствовать численные характеристики элементарных частиц и электронов — их массы, заряды разной природы, спины. Опыт последнего столетия убедительно показывает, что необходимо присутствие по крайней мере еще двух величин — постоянной Планка К и скорости света с. Постоянная Планка — мера корпускулярно-волнового дуализма микрочастиц. Скорость света — максимальная скорость распространения сигнала без нее не обходится ни одна формула релятивистской механики (см. гл. 4). [c.273]

    Позитрон — первая античастица. Физический вакуум. Когда Дирак (Поль Адриан Морис Дирак, 1902-1984 гг.. Нобелевская премия 1933 г.) сформулировал свое знаменитое уравнение (1930 г.), положившее начало релятивистской квантовой механике, возникло странное противоречие. Оказалось, что решения уравнения (состояния) могут описывать не только реальные электроны с положительной энергией , зависяш,ей от импульса р так, как предписывает релятивистская механика [е = (т с + с р ) / ). Они описывают и состояния с отрицательной энергией [е = — (т с + с р ) / ), которые, казалось бы, ничему реальному не соответствуют. Отмахнуться от них не было никакой возможности, хотя бы потому, что электрон с положительной энергией должен упасть в состояние с отрицательной энергией, излучив лишнюю энергию в виде световых квантов. Последнее всегда происходит, если атомный электрон оказывается в состоянии с высокой энергией и имеются свободные состояния с энергией пониже. [c.283]

    Подстановка выражений для плотностей потоков, выведенных в настоящем разделе, в уравнения сохранения из раздела 18.3 приводит к общим дифференциальным уравнениям в частных производных, описывающим движение многокомпонентной смеси, которое сопровождается теплообменом, массообменом и химическими реакциями. Слово общие всегда, конечно, необходимо применять с некоторой осторожностью, поскольку часто можно придумать более общие случаи. В качестве такого примера достаточно напомнить область магнитогидродинамики. Уравнения, описывающие многокомпонентные жидкие смеси, подвергнутые воздействию электромагнитного поля, представляют собой уравнения сохранения и уравнения электродинамики Максвелла. Эта область интересна в связи с астрофизическими явлениями, поведением ионизированного газа и струй плазмы [25—27]. Другая область, не охваченная нашими уравнениями, — область релятивистской механики жидкостей. Упомянутая область включает релятивистские эффекты, которые играют важную роль при скоростях жидкости, близких к скорости света [28]. [c.503]


    В релятивистской механике оно заменяется следуюш,им соотношением  [c.21]

    Действительно, согласно общим принципам релятивистской механики, импульс частицы равен ev/ , где е — ее полная энергия, включающая энергию покоя. Так как энергия взаимодействия электрона как с решеткой, так и с другими электронами значительно меньше энергии покоя, импульс электрона с большой точностью равен (шо — масса свободного электрона). Отсюда сразу следует выписанное выше соотношение. [c.211]

    Следует, однако, иметь в виду, что в предельном случае очень малых длин волн, когда де-бройлевская длина волны сравнима с размерами рассматриваемой частицы, квантовая механика переходит в свой классический прототип — релятивистскую механику или механику Ньютона (в нерелятивистс.ком случав). [c.183]

    Закон изменения масштаба (34) справедлив также в теории упругости, теории пластичности и в динамике взрывных процессов ) он назван законом Кранца. Вообще он справедлив всегда, когда тензор напряжений есть функция только от деформации и не зависит от ее скорости, и всякий раз, когда в некотором напряженном состоянии освобождается определенная (в расчете на единицу объема) химическая энергия, как это требуется в условиях Чепмена — Жуге ([6], 87). Любопытно, что этот закон справедлив также в релятивистской механике жидкостей. [c.147]

    Предварительные замечания. Начало XX в. ознаменовалось глубоким изменением основных представленйй классической физики, которое шло в двух направлениях. В 1905 г. Эйнштейн показал, что обычная механика, считавшаяся на протяжении сотен лет одной из незыблемых основ точных наук, на самом деле есть лишь предельный случай более общей релятивистской механики, основанной на теории относительности. На протяжении 10 лет эта [c.32]

    Зоммерфельд развил теорию Бора, введя в рассмотрение эллиптические орбиты для электронов, а позн е применил релятивистскую механику к движению электрона. Такая трактовка привела к более сложному выражению для энергии различных электронных орбит в атоме водорода и позволила предсказать, что для данного главного квантового числа имеется в действительности несколько термов с близкими энергиями. Эта уточненная теория предсказывает группу линий с несколько различающимися длинами волн — [c.29]

    Из регнепия уравнения Шредингера для атома Н получаются естественным образом три параметра. Эти величины, а также еще одна, получающаяся из релятивистской механики Дирака, называются квантовыми числами. Они играют чрезвычайно важную роль в нашем ионимании электронных переходов, ответственных за линейчатые спектры атомов, и представляют основу логического объяснения периодической системы элементов. [c.31]

    В классической физике и квантовой механике, включая и релятивистскую механику, время, как уже отмечалось, выступает лишь как внешний параметр, не имеющий выделенного направления. В классической и квантовой динамике нет ничего такого, что позволило бы отличить прошлое от будущего. Такую динамику Пригожин назвал физикой существующего, а современную, в значительной мере созданную им термодинамику, – физикой возникающего. Даже равновесная термодинамика уже содержит специальную функцию состояния – энтропию, наделяющую время определенным направлением. Энтропия – это стрела времени, устанавливающая различие между прошлым и будущим. Нелинейная термодинамика неравновесных процессов идет в этом отношении еще дальше и формирует новое представление о времени как внутренней переменной, присущей данной макроскопической системе и отражающей ее историю. Такое время в отличие от внешнего времени характеризует эволюционное состояние системы, пройденные ею этапы развития. Еще Аристотель различал время как “движение” (кинезис) и время как “рождение и гибель” (метаболе). Возраст живого организма может определяться астрономическим временем, а может – внутренним, биологическим временем, не совпадающим с внешним временем, хотя и выраженным в тех же единицах. Два организма одного вида, рожденные одновременно (даже однояйцевые близнецы), имеют одинаковый возраст в масштабе внешнего, астрономического времени и в зависимости от условий созревания могут иметь разный возраст в масштабе внутреннего времени, ха- [c.458]


Энергия и импульс

Страница 1 из 3

Импульсом частицы называется, как известно, вектор р=∂L/∂v (∂L/∂v — символическое обозначение вектора, компоненты которого равны производным от L по соответствующим компонентам v). С помощью (8.2) находим

p =                                                     (9.1)

При малых скоростях (v<<c) или в пределе при c→ это выражение переходит в классическое p=mv. При v=c импульс обращается в бесконечность.

Производная от импульса по времени есть сила, действующая на частицу. Пусть скорость частицы изменяется только по направлению, т.е. сила направлена перпендикулярно скорости. Тогда

=  .                                        (9.2)

Если же скорость меняется только по величине, т. е. сила направлена по скорости, то

   = .                                 (9.3)

Мы видим, что в обоих случаях отношение силы к ускорению различно.

Энергией  частицы называется величина

= pv − L.

Подставляя сюда выражения (8.2) и (9.1) для L и p, получим

= .                                               (9.4)

Эта очень важная формула показывает, в частности, что в релятивистской механике энергия свободной частицы не обращается в нуль при v=0, а остается конечной величиной, равной

= mc2.                                                             (9.5)

Ее называют энергией покоя частицы.

При малых скоростях (v<<c) имеем, разлагая (9.4) по степеням v/c:

 ≈ mc2 + ,

т. е. за вычетом энергии покоя классическое выражение для кинетической энергии частицы.

Подчеркнем, что хотя мы говорим здесь о «частице», но ее «элементарность» нигде не используется. Поэтому полученные формулы в равной степени применимы и к любому сложному телу, состоящему из многих частиц, причем под m надо понимать полную массу тела, а под v — скорость его движения как целого. В частности, формула (9.5) справедлива и для любого покоящегося как целое тела. Обратим внимание на то, что энергия свободного тела (т.е. энергия любой замкнутой системы) оказывается в релятивистской механике вполне определенной, всегда положительной величиной, непосредственно связанной с массой тела. Напомним в этой связи, что в классической механике энергия тела определена лишь с точностью до произвольной аддитивной постоянной, и может быть как положительной, так и отрицательной.

Энергия покоящегося тела содержит в себе, помимо энергий покоя входящих в его состав частиц, также кинетическую энергию частиц и энергию их взаимодействия друг с другом. Другими словами, mc2 не равно сумме ∑mαc2 (mα — массы частиц), а потому и m не равно ∑mα. Таким образом, в релятивистской механике не имеет места закон сохранения массы: масса сложного тела не равна сумме масс его частей. Вместо этого имеет место только закон сохранения энергии, в которую включается также и энергия покоя частиц.

Возводя выражения (9.1) и (9.4) в квадрат и сравнивая их, найдем следующее соотношение между энергией и импульсом частицы:

= p2 + m2c2.                                                (9.6)

Энергия, выраженная через импульс, называется, как известно, функцией Гамильтона :

= c .                                         (9.7)

релятивистских величин | Безграничная физика

Релятивистское сложение скоростей

Формула сложения скоростей – это уравнение, которое связывает скорости движущихся объектов в различных системах отсчета.

Цели обучения

Экспресс-формулы сложения скоростей для объектов, скорость которых намного меньше и приближается к скорости света

Основные выводы

Ключевые моменты
  • Когда два объекта движутся медленно по сравнению со скоростью света, достаточно точно использовать векторную сумму скоростей: [latex] \ text {s} = \ text {u} + \ text {v} [/ latex] .{2}} [/ латекс].
  • Закон композиции для скоростей дал первую проверку кинематики специальной теории относительности, когда с помощью интерферометра Майкельсона Ипполит Физо измерил скорость света в жидкости, движущейся параллельно свету.
Ключевые термины
  • специальная теория относительности : теория, которая (игнорируя эффекты гравитации) согласовывает принцип относительности с наблюдением, что скорость света постоянна во всех системах отсчета.
  • интерферометр : Любой из нескольких инструментов, которые используют интерференцию волн для определения длины волны и скорости волны, определения показателей преломления и измерения малых расстояний, изменений температуры, напряжений и многих других полезных измерений.
  • скорость света : скорость электромагнитного излучения в абсолютном вакууме: ровно 299 792 458 метров в секунду по определению

Формула сложения скоростей – это уравнение, которое связывает скорости движущихся объектов в различных системах отсчета.

Как Галилео Галилей заметил в 17 веке, если корабль движется относительно берега со скоростью [латекс] \ text {v} [/ latex], а муха движется со скоростью [латекс] \ text {u} [/ латекс], измеренный на корабле, вычисление скорости мухи, измеренное на берегу, – это то, что подразумевается под сложением скоростей [латекс] \ text {v} [/ latex] и [латекс] \ text {u} [/латекс]. Когда и муха, и корабль движутся медленно по сравнению со скоростью света, достаточно точно использовать векторную сумму [latex] \ text {s} = \ text {u} + \ text {v} [/ latex], где [latex] \ text {s} [/ latex] – скорость мухи относительно берега.

Согласно специальной теории относительности, корпус корабля имеет другую тактовую частоту и меру расстояния, и понятие одновременности в направлении движения изменено, поэтому изменен закон сложения скоростей.

Поскольку специальная теория относительности требует, чтобы скорость света была одинаковой во всех системах отсчета, свет, падающий спереди движущегося автомобиля, не может двигаться быстрее, чем свет от стационарного фонаря. Поскольку это противоречит тому, что Галилей использовал для добавления скоростей, необходим новый закон сложения скоростей.{2}} [/ латекс].

Закон композиции скоростей дал первую проверку кинематики специальной теории относительности. Используя интерферометр Майкельсона, Ипполит Физо измерил скорость света в жидкости, движущейся параллельно свету в 1851 году. Скорость света в жидкости ниже, чем скорость света в вакууме, и она изменяется, если жидкость движется вместе с ней. свет. Скорость света в коллинеарно движущейся жидкости точно предсказывается коллинеарным случаем релятивистской формулы.

Схема эксперимента Физо : световой луч, исходящий от источника S ‘, отражается светоделителем G и коллимируется в параллельный пучок линзой L.После прохождения щелей O1 и O2 через них проходят два луча света. трубки A1 и A2, по которым вода течет вперед и назад, как показано стрелками. Лучи отражаются от зеркала m в фокусе линзы L ’, так что один луч всегда распространяется в том же направлении, что и поток воды, а другой луч – противоположно направлению потока воды.Пройдя назад и вперед через трубки, оба луча объединяются в точке S, где они создают интерференционные полосы, которые можно визуализировать через иллюстрированный окуляр. Интерференционная картина может быть проанализирована, чтобы определить скорость света, движущегося по каждому участку трубки.

Релятивистский импульс

Релятивистский импульс задается как [latex] \ gamma \ text {m} _ {0} \ text {v} [/ latex], где [latex] \ text {m} _ {0} [/ latex] – инвариант объекта. масса и [латекс] \ гамма [/ латекс] – это преобразование Лоренца.

Цели обучения

Сравните ньютоновские и релятивистские импульсы для объектов, скорость которых намного меньше и приближается к скорости света

Основные выводы

Ключевые моменты
  • Ньютоновская физика предполагает, что абсолютное время и пространство существуют вне любого наблюдателя, в результате чего предсказывается, что скорость света может изменяться от одной системы отсчета к другой.
  • В специальной теории относительности уравнения движения не зависят от системы отсчета, в то время как скорость света (c) инвариантна.
  • В рамках классической механики релятивистский импульс очень близок к ньютоновскому. При низкой скорости [латекс] \ gamma \ text {m} _ {0} \ text {v} [/ latex] примерно равен [latex] \ text {m} _ {0} \ text {v} [/ latex ], ньютоновское выражение для импульса.
Ключевые термины
  • Преобразование Галилея : преобразование, используемое для преобразования между координатами двух систем отсчета, которые отличаются только постоянным относительным движением в рамках построений ньютоновской физики.
  • Преобразование Лоренца : преобразование, связывающее пространственно-временные координаты одной системы отсчета с другой в специальной теории относительности
  • специальная теория относительности : теория, которая (игнорируя эффекты гравитации) согласовывает принцип относительности с наблюдением, что скорость света постоянна во всех системах отсчета.

Релятивистский импульс

Ньютоновская физика предполагает, что абсолютное время и пространство существуют вне любого наблюдателя.Это приводит к теории относительности Галилея, которая утверждает, что законы движения одинаковы во всех инерциальных системах отсчета. Это также приводит к предсказанию того, что скорость света может изменяться от одной системы отсчета к другой. Однако это противоречит наблюдениям. В специальной теории относительности Альберт Эйнштейн придерживается постулата о том, что уравнения движения не зависят от системы отсчета, но предполагает, что скорость света c инвариантна. В результате положение и время в двух системах отсчета связаны преобразованием Лоренца вместо преобразования Галилея.

Альберт Эйнштейн : Альберт Эйнштейн в 1921 году

Рассмотрим, например, систему отсчета, движущуюся относительно другой со скоростью v в направлении x . Преобразование Галилея дает координаты движущейся системы отсчета как

[латекс] \ text {t} ‘= \ text {t} [/ latex]

[латекс] \ text {x} ‘= \ text {x} – \ text {vt} [/ latex]

, а преобразование Лоренца дает

[латекс] \ text {t} ‘= \ gamma (\ text {t} – \ frac {\ text {vx}} {\ text {c} ^ {2}}) [/ latex]

[латекс] \ text {x} ‘= \ gamma (\ text {x} – \ text {vt}) [/ latex]

где [латекс] \ гамма [/ латекс] – коэффициент Лоренца:

[латекс] \ gamma = \ frac {1} {\ sqrt {1 – (\ text {v} / \ text {c}) ^ {2}}} [/ latex].

Законы сохранения в физике, такие как закон сохранения количества движения, должны быть инвариантными. То есть свойство, которое необходимо сохранить, должно оставаться неизменным независимо от изменений условий измерения. Это означает, что закон сохранения должен выполняться в любой системе отсчета. Второй закон Ньютона [с массой, фиксированной в выражении для количества движения (p = m * v)] не инвариантен относительно преобразования Лоренца. Однако его можно сделать инвариантным, сделав инерциальную массу объекта м функцией скорости:

[латекс] \ text {m} = \ gamma \ text {m} _ {0} [/ latex]

где [latex] \ text {m} _ {0} [/ latex] – инвариантная масса объекта.

Модифицированный импульс,

[латекс] \ text {p} = \ gamma \ text {m} _ {0} \ text {v} [/ latex]

подчиняется второму закону Ньютона:

[латекс] \ text {F} = \ frac {\ text {dp}} {\ text {dt}} [/ latex].

Важно отметить, что для скоростей, намного меньших скорости света, ньютоновский импульс и релятивистский импульс примерно одинаковы. Однако по мере приближения к скорости света релятивистский импульс становится бесконечным, в то время как ньютоновский импульс продолжает линейно возрастать.Таким образом, необходимо использовать выражение для релятивистского импульса, когда мы имеем дело со скоростями, близкими к скорости света.

Релятивистский и ньютоновский импульс : Этот рисунок показывает, что релятивистский импульс приближается к бесконечности по мере приближения к скорости света. Ньютоновский импульс линейно увеличивается со скоростью.

Релятивистская энергия и масса

В специальной теории относительности, когда объект приближается к скорости света, энергия и импульс объекта неограниченно возрастают.{2}}} [/ латекс].

Ключевые термины
  • Фактор Лоренца : Фактор, используемый в специальной теории относительности для вычисления степени замедления времени, сокращения длины и релятивистской массы объекта, движущегося относительно наблюдателя.
  • масса покоя : масса тела, когда оно не движется относительно наблюдателя
  • специальная теория относительности : теория, которая (игнорируя эффекты гравитации) согласовывает принцип относительности с наблюдением, что скорость света постоянна во всех системах отсчета.

Релятивистская энергия и масса

Согласно специальной теории относительности, объект, обладающий массой, не может двигаться со скоростью света. Когда объект приближается к скорости света, его энергия и импульс неограниченно возрастают. Релятивистские поправки на энергию и массу необходимо делать из-за того факта, что скорость света в вакууме постоянна во всех системах отсчета. Сохранение массы и энергии – общепринятые законы физики. Чтобы эти законы выполнялись во всех системах отсчета, необходимо применить специальную теорию относительности.Важно отметить, что для объектов со скоростями, которые значительно ниже скорости света, выражения для релятивистской энергии и массы дают значения, примерно равные их ньютоновским аналогам. 2}} [/ latex], где v – относительная скорость между инерциальными системами отсчета, c – скорость света.

Ричард Толмен и Альберт Эйнштейн : Ричард Толмен (1881-1948) с Альбертом Эйнштейном (1879-1955) в Калтехе, 1932 год

Когда относительная скорость равна нулю, просто равна 1, а релятивистская масса сводится к массе покоя. По мере того, как скорость увеличивается до скорости света (c), знаменатель правой части приближается к нулю и, следовательно, приближается к бесконечности.

В формуле для количества движения возникающая масса является релятивистской массой.{2}} [/ латекс]. Здесь термин представляет собой квадрат евклидовой нормы (общей длины вектора) различных векторов импульса в системе, который сводится к квадрату простой величины импульса, если рассматривать только одну частицу. Это уравнение сводится к моменту, когда импульс равен нулю. Для фотонов, для которых уравнение сводится к.

Сегодня предсказания релятивистской энергии и массы обычно подтверждаются экспериментальными данными ускорителей частиц, таких как коллайдер релятивистских тяжелых ионов.Увеличение релятивистского импульса и энергии не только точно измеряется, но также необходимо для понимания поведения циклотронов и синхротронов, которые ускоряют частицы почти до скорости света.

Материя и антивещество

Антивещество состоит из античастиц, которые имеют ту же массу, что и частицы обычного вещества, но имеют противоположный заряд и квантовый спин.

Цели обучения

Описание свойств античастиц

Основные выводы

Ключевые моменты
  • Конечным результатом встречи антивещества с веществом является выделение энергии, пропорциональной массе, как показано в уравнении эквивалентности массы и энергии: E = mc 2 .
  • Хотя кажется, что почти вся материя, наблюдаемая с Земли, состоит из материи, а не антивещества, антивещество все еще может существовать в относительно больших количествах в далеких галактиках.
  • Никакого объяснения кажущейся асимметрии вещества и антивещества в видимой Вселенной не существует.
Ключевые термины
  • аннигиляция : процесс объединения частицы и соответствующей ей античастицы для получения энергии
  • позитрон : Эквивалент электрона в антивеществе, имеющий ту же массу, но положительный заряд.
  • антивещество : вещество, состоящее из античастиц тех, что составляют нормальную материю

Антивещество – это материал, состоящий из античастиц, которые имеют ту же массу, что и частицы обычного вещества, но имеют противоположный заряд и квантовый спин. Античастицы связываются друг с другом, образуя антивещество так же, как нормальные частицы связываются, образуя нормальное вещество. Например, позитрон (античастица электрона, символ e + ) и антипротон (символ p ) могут образовывать атом антиводорода.Более того, смешивание вещества и антивещества может привести к аннигиляции и того, и другого точно так же, как смешивание античастиц и частиц. Это порождает фотоны высокой энергии (гамма-лучи) и другие пары частица-античастица. Конечным результатом встречи антивещества с веществом является выделение энергии, пропорциональной массе, как показано в уравнении эквивалентности массы и энергии: E = mc 2 .

Антиводород и атомы водорода : Антиводород состоит из антипротона и позитрона; водород состоит из протона и электрона.

Кажется, что почти вся материя, наблюдаемая с Земли, состоит из материи, а не из антивещества. Если бы существовали области пространства с преобладанием антивещества, гамма-лучи, образующиеся в реакциях аннигиляции вдоль границы между областями материи и антивещества, можно было бы обнаружить.

Антивещество может все еще существовать в относительно больших количествах в далеких галактиках из-за космической инфляции в изначальное время Вселенной. Ожидается, что галактики на основе антивещества, если они существуют, будут иметь такой же химический состав, спектры поглощения и излучения, что и галактики с нормальной материей, а их астрономические объекты будут идентичны с точки зрения наблюдений, что затрудняет их отличия от галактик с нормальной материей. {2} [ /латекс].

  • На низкой скорости ([latex] \ text {v} << \ text {c} [/ latex]) релятивистская кинетическая энергия может быть аппроксимирована классической кинетической энергией. Таким образом, полную энергию можно разделить на энергию массы покоя плюс традиционную ньютоновскую кинетическую энергию на малых скоростях.
  • Ключевые термины
    • специальная теория относительности : теория, которая (игнорируя эффекты гравитации) согласовывает принцип относительности с наблюдением, что скорость света постоянна во всех системах отсчета.
    • классическая механика : Все физические законы природы, которые объясняют поведение нормального мира, но нарушаются при работе с очень маленькими (см. Квантовую механику) или очень быстрыми или очень тяжелыми (см. Теорию относительности).
    • Фактор Лоренца : Фактор, используемый в специальной теории относительности для вычисления степени замедления времени, сокращения длины и релятивистской массы объекта, движущегося относительно наблюдателя.

    В классической механике кинетическая энергия объекта зависит от массы тела, а также от его скорости.{2}} {2 \ text {m}} [/ latex],

    где [латекс] \ text {p} [/ latex] – импульс.

    Если скорость тела составляет значительную долю скорости света, необходимо использовать специальную теорию относительности для расчета его кинетической энергии. Важно знать, как применить специальную теорию относительности к проблемам с высокоскоростными частицами. В специальной теории относительности мы должны изменить выражение для количества движения. Используя [latex] \ text {m} [/ latex] для массы покоя, [latex] \ text {v} [/ latex] и [latex] \ nu [/ latex] для скорости и скорости объекта соответственно, и [latex ] \ text {c} [/ latex] для скорости света в вакууме релятивистское выражение для количества движения:

    Журнал «Тайм» – 1 июля 1946 г. : Популярная связь между Эйнштейном, E = mc2, и атомной бомбой была заметно обозначена на обложке журнала «Тайм» (июль 1946 г. 2 [/ latex].

    Таким образом, полную энергию можно разделить на энергию массы покоя плюс традиционную классическую кинетическую энергию на малых скоростях.

    Релятивистская механика – обзор

    В релятивистской механике тензор углового момента определяется выражением

    (15.58) Mnk = xnpk − xkpn (k, n = 0,1,2,3).

    Только пространственные компоненты тензора углового момента с ( k , n = 1,2,3) имеют физический смысл и совпадают с обычным определением углового момента в нерелятивистской механике.В нерелятивистской механике принято формировать осевой трехмерный вектор углового момента

    (15.59) Mν = 12eναβMαβ = eναβxαpβ (α, β = 1,2,3)

    или в классической векторной записи

    (15.60) M → = 1 → 2 → 3 → x1x2x3p1p2p3 = r → × p →.

    Закон сохранения тензора углового момента (15.58) является следствием изотропной природы четырехмерного пространства-времени. Трехмерный закон сохранения углового момента является следствием изотропной природы трехмерного пространства.Из-за изотропной природы четырехмерного пространства-времени механические свойства свободной частицы остаются неизменными после вращений в четырехмерном пространстве-времени. Давайте рассмотрим частный случай поворота на некоторый угол θ вокруг 3-х осей в трехмерном пространстве. Тогда связь между координатами z m в повернутой инерциальной системе отсчета K ′ и координатами x k в исходной инерциальной системе отсчета K задается как

    (15.61) zn = Ωjnxj (j, n = 0,1,2,3).

    Используя (5.40), мы можем переписать (15.61) в матричной форме

    (15.62) z0z1z2z3 = 10000cosθsinθ00 − sinθcosθ00001x0x1x2x3.

    Если мы теперь рассмотрим поворот на некоторый бесконечно малый угол δθ ≈ 0, то у нас будет

    (15,63) cosδθ≈1, sinδθ≈δθ.

    Подставляя (15,63) в (15,62), мы получаем

    (15,64) z0z1z2z3 = 100001δθ00 − δθ100001x0x1x2x3,

    или

    (15,65) zn = xn + δΩjnxΩ δjn + xnkj = xn + xn + xn = xn + xn = xn = xn смешанный тензор, определяемый следующей антисимметричной матрицей:

    (15.66) δΩjn = 000000δθ00 − δθ000000.

    Используя здесь (13.9) и (13.12), мы получаем ковариантные координаты x k четырехмерного пространства-времени в матричной форме

    (15,67) xk = gjkxj = 10000−10000−10000 −1x0x1x2x3 знак равно x0 − x1 − x2 − x3.

    Мы также можем вычислить компоненты антисимметричного контравариантного тензора δ Ω nk в матричной форме следующим образом:

    (15,68) δΩnk = gjkδΩjn,

    или

    (15.69) δΩnk = 10000−10000−10000−1000000δθ00 − δθ000000.

    Таким образом, окончательно получаем матричную форму антисимметричного контравариантного тензора δ Ω nk следующим образом:

    (15.70) δΩnk = 000000 − δθ00δθ000000.

    Для произвольного бесконечно малого вращения контравариантный тензор δ Ω nk имеет более сложную форму по сравнению с простой матрицей (15.70), но это всегда антисимметричный тензор. Таким образом, у нас всегда

    (15.71) δΩnk = −δΩkn (k, n = 0,1,2,3),

    и наиболее общий бесконечно малый поворот координат задается следующими соотношениями преобразования:

    (15,72) zn = xn + δxn, δxn = δΩnkxk.

    Наиболее общее бесконечно малое вращение четырехскоростного вектора задается следующими соотношениями преобразования:

    (15.73) u′n = un + δun, δun = δΩnkuk.

    Функция Лагранжа частицы L ( x n , u n ) должна быть инвариантной относительно этого бесконечно малого вращения, то есть должно быть δL = 0.Теперь вычислим изменение лагранжиана L относительно бесконечно малых вращений (15.72) и (15.73), то есть

    (15.74) δL = ∂L∂xnδxn + ∂L∂unδun = 0,

    или

    (15,75) δL = ∂L∂xnδΩnkxk + ∂L∂unδΩnkuk = 0.

    Подставляя уравнения движения (15.2) в (15.75), получаем

    (15.76) δL = δΩnkdds∂L∂unxk + ∂L∂unuk = 0.

    Используя определение четырехвектора энергии-импульса (15.38) и определение четырехскорости (13.34), получаем

    (15,77) δL = −δΩnkdpndsxk + pndxkds = −ddsδΩnkpnxk = 0.

    Используя здесь антисимметрию тензора δ Ω nk , (15.77) становится

    (15.78) δL = −12δΩnkddspnxk − pkxn = 12δΩnkdMnkds = 0.

    Из (15.78), как прямое следствие изотропности пространства-времени, получаем закон сохранения тензора углового момента, то есть

    (15.79) dMnkds = 0⇒dMnkdt = 0⇒Mnk = Константа

    Пространственная часть этого закона сохранения для ( n , k = 1,2,3), то есть

    (15.80) Mν = eναβxαpβ = Константа

    – обычный закон сохранения трехмерного вектора углового момента.

    Специальная теория относительности Эйнштейна

    Специальная теория относительности объясняет, как пространство и время связаны между собой для объектов, движущихся с постоянной скоростью по прямой. Один из самых известных его аспектов касается объектов, движущихся со скоростью света.

    Проще говоря, когда объект приближается к скорости света, его масса становится бесконечной, и он не может двигаться быстрее света.Это космическое ограничение скорости было предметом многочисленных дискуссий в физике и даже в научной фантастике, поскольку люди думают о том, как путешествовать на огромные расстояния.

    Специальная теория относительности была разработана Альбертом Эйнштейном в 1905 году и составляет часть основы современной физики. Закончив свою работу по специальной теории относительности, Эйнштейн десять лет размышлял о том, что произойдет, если ввести ускорение. Это легло в основу его общей теории относительности, опубликованной в 1915 году.

    История

    До Эйнштейна астрономы (по большей части) понимали Вселенную в терминах трех законов движения, представленных Исааком Ньютоном в 1686 году.Вот эти три закона:

    (1) Объекты в движении (или в состоянии покоя) остаются в движении (или в состоянии покоя), если внешняя сила не вызывает изменения.

    (2) Сила равна изменению количества движения за изменение времени. Для постоянной массы сила равна массе, умноженной на ускорение.

    (3) Для каждого действия существует равная и противоположная реакция.

    Но, согласно Британской энциклопедии, за десятилетия до появления Эйнштейна в теории были трещины. В 1865 году шотландский физик Джеймс Клерк Максвелл продемонстрировал, что свет представляет собой волну, имеющую как электрические, так и магнитные компоненты, и установил скорость света (186 000 миль в секунду).Ученые предположили, что свет должен проходить через некую среду, которую они назвали эфиром. (Теперь мы знаем, что передающая среда не требуется и что свет в космосе движется в вакууме.)

    Двадцать лет спустя неожиданный результат поставил это под сомнение. Физик А.А. Майкельсон и химик Эдвард Морли (оба в то время американцы) вычислили, как движение Земли через этот «эфир» влияет на то, как измеряется скорость света, и обнаружили, что скорость света одинакова независимо от движения Земли.Это привело к дальнейшим размышлениям австрийского физика Эрнста Маха и французского математика Анри Пуанкаре о поведении света и его несовпадении с классической механикой.

    Эйнштейн начал думать о поведении света, когда ему было всего 16 лет, в 1895 году. Он провел мысленный эксперимент, говорится в энциклопедии, где он ездил на одной световой волне и смотрел на другую световую волну, движущуюся параллельно ему.

    Классическая физика должна сказать, что световая волна, на которую смотрел Эйнштейн, будет иметь относительную скорость, равную нулю, но это противоречило уравнениям Максвелла, которые показали, что свет всегда имеет одинаковую скорость: 186 000 миль в секунду.Другая проблема с относительными скоростями заключается в том, что они могут показать, что законы электромагнетизма меняются в зависимости от вашей точки обзора, что также противоречило классической физике (которая гласила, что законы физики одинаковы для всех). специальная теория относительности, которую он разбил на повседневный пример человека, стоящего у движущегося поезда и сравнивающего наблюдения с человеком внутри поезда. Он представил себе поезд, находящийся на пути между двумя деревьями.Если молния ударит по обоим деревьям одновременно из-за движения поезда, человек в поезде увидит, как молния поразит одно дерево раньше другого. Но человек рядом с трассой видел бы одновременные удары.

    «Эйнштейн пришел к выводу, что одновременность относительна; события, одновременные для одного наблюдателя, могут не быть для другого», – говорится в энциклопедии. “Это привело его к парадоксальной идее о том, что время течет по-разному в зависимости от состояния движения, и к выводу, что расстояние также относительно.”

    Известное уравнение

    Работа Эйнштейна привела к некоторым поразительным результатам, которые сегодня все еще кажутся нелогичными на первый взгляд, хотя его физика обычно вводится на уровне средней школы.

    В 2015 году исполняется 100 лет со дня публикации книги Альберта Эйнштейна« Генерал ». Теория относительности. Изучите основы теории относительности Эйнштейна в нашей инфографике. (Изображение предоставлено Карлом Тейтом, художником по инфографике)

    Одно из самых известных уравнений в математике исходит из специальной теории относительности.Уравнение – E = mc 2 – означает «энергия равна массе, умноженной на квадрат скорости света». Это показывает, что энергия ( E ) и масса ( m ) взаимозаменяемы; это разные формы одного и того же. Если масса каким-то образом полностью преобразована в энергию, это также показывает, сколько энергии будет находиться внутри этой массы: довольно много. (Это уравнение является одной из демонстраций того, почему атомная бомба настолько мощна, если ее масса преобразована во взрыв.)

    Это уравнение также показывает, что масса увеличивается со скоростью, что фактически ограничивает скорость движения объектов во Вселенной. Проще говоря, скорость света ( c ) – это самая высокая скорость, с которой объект может перемещаться в вакууме. По мере движения объекта его масса также увеличивается. Приближаясь к скорости света, масса настолько велика, что достигает бесконечности, и для ее перемещения потребуется бесконечная энергия, что ограничивает скорость движения объекта. Единственная причина, по которой свет движется с такой скоростью, заключается в том, что фотоны, квантовые частицы, из которых состоит свет, имеют нулевую массу.

    Особая ситуация во вселенной малых, называемая «квантовой запутанностью», сбивает с толку, потому что она, кажется, включает квантовые частицы, взаимодействующие друг с другом на скоростях, превышающих скорость света. В частности, измерение свойства одной частицы может мгновенно определить свойства другой частицы, независимо от того, как далеко они находятся. Об этом явлении написано много, но до сих пор оно не получило полного объяснения с точки зрения выводов Эйнштейна.

    Другой странный вывод работы Эйнштейна связан с осознанием того, что время движется относительно наблюдателя.Движущийся объект испытывает замедление времени, что означает, что время движется медленнее, когда человек движется, чем когда он стоит на месте. Следовательно, движущийся человек стареет медленнее, чем человек в состоянии покоя. Так что да, когда астронавт Скотт Келли провел почти год на борту Международной космической станции в 2015–16 годах, его брат-близнец, астронавт Марк Келли, постарел немного быстрее, чем Скотт.

    Это становится особенно очевидным при скоростях, приближающихся к скорости света. Представьте себе 15-летнего подростка, путешествующего в 99 лет.5 процентов скорости света в течение пяти лет (с точки зрения космонавта). По данным НАСА, когда 15-летний мальчик вернется на Землю, ему будет всего 20 лет. Его одноклассникам, однако, будет 65 лет.

    Хотя на этот раз расширение времени звучит очень теоретически, у него есть и практическое применение. Если у вас в автомобиле есть приемник глобального позиционирования (GPS), он пытается найти сигналы как минимум от трех спутников для координации вашего местоположения. Спутники GPS отправляют синхронизированные радиосигналы, которые прослушивает приемник, триангулируя (или, точнее говоря, трилатерируя) свое положение на основе времени прохождения сигналов.Проблема в том, что атомные часы на GPS движутся и, следовательно, будут работать быстрее, чем атомные часы на Земле, что создает проблемы с синхронизацией. Таким образом, по словам Ричарда Погга, астронома из Университета штата Огайо, инженерам необходимо сделать часы на GPS медленнее.

    По данным Physics Central, часы в космосе идут быстрее, потому что спутники GPS находятся над Землей и испытывают более слабую гравитацию. Таким образом, даже несмотря на то, что спутники GPS движутся и каждый день испытывают замедление на семь микросекунд из-за своего движения, в результате более слабой гравитации часы тикают примерно на 45 микросекунд быстрее, чем наземные часы.Если сложить эти два показателя, то спутниковые часы GPS будут тикать быстрее, чем наземные часы, примерно на 38 микросекунд в день.

    Специальная теория относительности и квантовая механика

    По мере того, как наши знания физики расширяются, ученые сталкиваются с более противоречивыми ситуациями. Один пытается согласовать общую теорию относительности, которая хорошо описывает то, что происходит с большими объектами, с квантовой механикой, которую лучше всего использовать для очень маленьких вещей (таких как распад атома урана).Эти две области, которые превосходно описывают свои отдельные области, несовместимы друг с другом, что расстроило Эйнштейна и поколения ученых после него.

    «Теория относительности дает бессмысленные ответы, когда вы пытаетесь масштабировать ее до квантового размера, в конечном итоге опускаясь до бесконечных значений в описании гравитации. Точно так же квантовая механика сталкивается с серьезными проблемами, когда вы увеличиваете ее до космических размеров», – статья в The Guardian указала в 2015 году.

    «Квантовые поля несут определенное количество энергии даже в кажущемся пустом пространстве, и количество энергии увеличивается по мере того, как поля становятся больше.Согласно Эйнштейну, энергия и масса эквивалентны (это сообщение E = mc 2 ), поэтому накопление энергии в точности похоже на накопление массы. Достаточно большой, и количество энергии в квантовых полях станет настолько большим, что создаст черную дыру, которая заставляет Вселенную складываться сама по себе. Упс. “

    Есть несколько идей по преодолению этого (которые выходят за рамки данной статьи), но один из подходов – представить квантовую теорию гравитации, в которой безмассовая частица (называемая гравитоном) генерирует силу.Но, как отметил физик Дэйв Голдберг в io9 в 2013 году, с этим есть проблемы. В самых маленьких масштабах гравитоны имели бы бесконечную плотность энергии, создавая невообразимо мощное гравитационное поле. Потребуются дополнительные исследования, чтобы увидеть, возможно ли это.

    Релятивистская механика

    Релятивистская механика

    Релятивистская механика


    Новую теорию механики пришлось свести к старой в пределе «малой скорости».

    Примеры — Что это за пример



    Время в кадре мгновенного покоя движущегося объекта – это время, сохраненное по часам на объекте – собственное время .

    Масса – это сопротивление ускорению движущегося объекта в его собственной системе мгновенного покоя . Эту величину часто называют «массой покоя».

    4-скорость объекта – это скорость изменения его пространственно-временного положения относительно его собственное время.

    Используйте определение импульса Ньютона: p = mv , но с m масса покоя и v 4-скорость. Результирующий объект состоит из четырех компонентов и называется четырехимпульсным .

    Примеры — Что это за пример


    Для релятивистского выражения

    Для ньютоновского выражения

    Релятивистское выражение дает более высокую ценность.

    Обратите внимание, что релятивистское выражение для К.Е. увеличивается без ограничений по мере приближения к скорости света.



    Предположим, что масса-энергия покоя частицы равна одному Мэв (независимо от того, что такое , что и ). Найди энергия, необходимая для ускорения частицы до 4/5 скорости света.

    Примеры — Что это за пример



    Две частицы, каждая с массой-энергией покоя 1 Мэв, сталкиваются лицом к лицу со скоростью 4/5 скорости света.Кинетическая энергия столкновения частично преобразуется в новые частицы того же типа. Сколько частиц такого типа можно было создать?

    Примеры — Что это за пример


    Откуда взялось E = mc

    2 ? Энергия в механике Ньютона и теории относительности Согласно теории относительности, законы физики могут быть одинаковыми для двух наблюдателей, движущихся относительно друг друга, но иногда они кажутся незнакомыми тем из нас, кто привык считать кинетическую энергию = ½ mv 2 .В этой анимации ракетный двигатель работает с постоянной скоростью, то есть вырабатывает постоянную мощность. (Это не обычное поведение для ракет, но это означает, что мы можем видеть, как увеличение кинетической энергии влияет на скорость и импульс в рамках классической и релятивистской механики.) Мы обсудим эту анимацию ниже.

    Другой способ записать второй закон Ньютона в классической механике – это теорема об энергии работы. Путем интегрирования равнодействующей силы F, действующей на тело, относительно расстояния, на которое перемещается центр масс,

      Ваш браузер не поддерживает видео тег.

    можно получить важный результат, заключающийся в том, что работа, выполняемая F, равна изменению величины ½mv 2 , величины, которую мы называем кинетической энергией. Это теорема. Как и следовало ожидать, в теории относительности все немного сложнее. Начнем со второго закона Ньютона, который гласит, что сила равна скорости изменения количества движения во времени:

      Мы интегрируемся для выполнения работы:
        Теперь нам нужно несколько выражений для подстановки:
          Нам нужно интегрировать как по v, так и по γ, поэтому полезно написать:
            и замена в нашем подынтегральном выражении дает:
              Отсюда мы получаем выражение для кинетической энергии K в специальной теории относительности:
                Это очень важное выражение, поэтому мы должны проверить, что на малых скоростях оно соответствует классическому выражению.Использование расширения, действительного для малых v / c:
                  так
                    как показано на графике справа – фиолетовая линия – это mc 2 + ½mv 2 . Ах, это обнадеживает. Но он также показывает нам ограничение скорости Эйнштейна. В классическом выражении v пропорционально квадратному корню из K. В принципе, v может быть сколь угодно большим. Однако в релятивистской механике (γ – 1) линейно увеличивается с кинетической энергией.И γ стремится к бесконечности, когда v стремится к c. Следовательно, конечное количество энергии никогда не может разогнать тело до скорости света .

                    Итак, теперь давайте снова посмотрим на уравнение (*):

                      Откуда взялась цифра 1 справа? Это начальное значение интеграла. Теперь все члены в этом уравнении – это энергии. Когда γ> 1, мы имеем ненулевую кинетическую энергию. Итак, если мы подумаем о γmc 2 как о полной энергии тела и напишем
                        тогда (γ – 1) * mc 2 – кинетическая энергия, а 1 * mc 2 – энергия, которой обладает тело при v = 0 и γ = 1.

                        Запомните надлежащее время и надлежащую длину модуля 4. Это время и длина тела, измеренные в его собственном корпусе. Чтобы мы могли написать

                          где E 0 = mc 2 – собственная энергия тела – энергия, которой оно обладает, даже когда оно не движется. Итак, это знаменитое уравнение E 0 = mc 2 возникло в результате определения постоянной интегрирования в релятивистской версии теоремы работы-энергии .
                            Учитывая, что это такое важное уравнение, интересно спросить: является ли это определение правильной энергии производным или вдохновенным предположением Эйнштейна? Мое мнение было бы где-то посередине.
                          Мы привыкаем к большим числам, но давайте вспомним, что
                            c 2 = 9,0 x 10 16 м 2 s -2 = 9,0 x 10 16 Дж / кг.
                          Да, c 2 имеет единицы джоули на килограмм, как мы можем видеть из знакомого выражения K classic = ½mv 2 .Таким образом, преобразование массы в один килограмм высвободило бы почти 10 17 джоулей. Ой! (Между прочим, эта функция не ограничивается ядерной энергией: см. Обсуждение энергии связи и пример ниже.)
                            Электростанция средней мощности может обеспечивать мощность 1 ГВт или 10 9 джоулей в секунду. Для обеспечения этой мощности непрерывно в течение года требуется 3 x 10 16 джоулей. Таким образом, на такой электростанции ежегодно исчезало бы около 300 граммов вещества.(Поскольку это небольшая сумма, фактический учет будет осложнен рядом процессов, происходящих со стержнями.)

                            На угольной электростанции с такой же постоянной мощностью исчезло бы 300 граммов вещества. Да, если бы вы могли собрать весь CO2, H2O, сажу, нитриды и другие продукты электростанции (ах, если бы мы только могли их собрать!), То у нас было бы на 300 граммов меньше, чем угля и воздуха, в которых мы реагировали. печи.

                            В качестве другого примера рассмотрим бомбу, уменьшающую общую массу топлива.Предположим, что взрыв этой бомбы отталкивает атмосферу в полусфере радиусом R. Тест. Простой подсчет, который можно проделать в уме, дает выход:

                              Работа, проделанная против атмосферы, равна атмосферному давлению, умноженному на 2πR 3 /3, объем полусферы.Предположим, что R равно 100 м, подстановка показывает, что для этого требуется потеря 2 микрограммов вещества. Опять же, неважно, химическая это или ядерная бомба. К счастью, очень сложно доставить очень большие химические бомбы. Я с сожалением должен сказать, что ядерные бомбы, достаточно маленькие, чтобы их можно было носить в рюкзаке или стрелять из пушки, но с огромной разрушительной силой, были разработаны несколькими странами.

                            Теперь рассмотрим двух наших путешественников.
                              Ваш браузер не поддерживает видео тег.

                            Обратите внимание на ограничение скорости Эйнштейна: космический корабль «Ньютон» может ускоряться бесконечно, но СС «Эйнштейн» приближается к c, поскольку вложенная в него энергия стремится к бесконечности. С другой стороны, по теории относительности импульс растет быстрее с вложенной энергией.

                            Для получения дополнительной информации о механике Ньютона и Галилея см. Physclips: Механика с анимацией и видеоклипами.

                          5.1: Различие между релятивистской и нерелятивистской квантовой механикой

                          Одним из ключевых моментов в физике частиц является то, что специальная теория относительности играет ключевую роль.2} \ psi. \ label {KG} \]

                          Это отличное уравнение для частиц без спина или частиц с единичным спином (бозонов), но не для описания фермионов (полуцелого спина), поскольку в этом уравнении нет информации о спине. Это требует внимательного рассмотрения, поскольку спин должен быть неотъемлемой частью релятивистского уравнения!

                          Дирак понял, что есть способ определить квадратный корень из оператора. Уловка, которую он использовал, состояла в том, чтобы определить четыре матрицы \ (\ alpha \), \ (\ beta \), каждая из которых имеет свойство, что их квадрат равен единице и что они антикоммутируются,

                          \ [\ begin {align} {2} \ alpha_i \ alpha_i & = I, & \ quad \\ [4pt] \ beta \ beta & = I \\ [4pt] \ alpha_i \ beta + \ beta \ alpha_i & = 0 \\ [4pt] \ alpha_i \ alpha_j + \ alpha_j \ alpha_i & = 0 \ quad \ text {$ i \ neq j $}.2 + c {\ vec {\ alpha}} \ cdot {\ vec {p}}) \ Psi = i \ hbar \ frac {\ partial} {\ partial t} \ Psi \]

                          Обратите внимание, что минимальная размерность матриц, в которой мы можем удовлетворить все условия, равна \ (4 \), и, следовательно, \ (\ Psi \) является четырехвектором! Это тесно связано с тем, что у этих частиц есть спин.

                          Давайте исследуем это уравнение немного подробнее. Одна из возможных форм \ (\ alpha_i \) и \ (\ beta \) –

                          \ [\ alpha_i = \ begin {pmatrix} 0 & \ sigma_i \\\ sigma_i & 0 \ end {pmatrix}, \ quad \ beta = \ begin {pmatrix} I & 0 \\ 0 & -I \ end {pmatrix }, \]

                          , где \ (\ sigma_i \) – спиновые матрицы Паули два на два.

                          \ [\ sigma_1 = \ begin {pmatrix} 0 & 1 \\ 1 & 0 \ end {pmatrix}, \ quad \ sigma_2 = \ begin {pmatrix} 0 & -i \\ i & 0 \ end {pmatrix}, \ quad \ sigma_3 = \ begin {pmatrix} 1 & 0 \\ 0 & -1 \ end {pmatrix}.2) \ end {pmatrix}, \]

                          с аналогичными выражениями для двух собственных векторов решений с отрицательной энергией. В пределе малого импульса собственные векторы с положительной энергией равны

                          \ [\ begin {pmatrix} 1 \\ 0 \\ 0 \\ 0 \ end {pmatrix} \ text {и} \ begin {pmatrix} 0 \\ 1 \\ 0 \\ 0 \ end {pmatrix}, \]

                          и, кажется, обозначают частицу со спином вверх и вниз. Мы покажем, что два других решения связаны с появлением античастиц (позитронов).

                          Так же, как фотоны – лучший способ анализировать (разлагать) электромагнитное поле, электроны и позитроны – естественный способ разложить поле Дирака, которое является общим решением уравнения Дирака.Такой анализ решения с точки зрения содержащихся в нем частиц называется (неправильно по историческим причинам) «вторым квантованием» и просто означает, что существует естественная основа, на которой мы можем сказать, что существует состояние с энергией \ (E \ ), который либо полон, либо пуст. Это можно было бы более правильно назвать «представлением числа заполнения», которое должно быть знакомо из физики конденсированного состояния. Это помогает нам увидеть, как частицу можно описать этими волновыми уравнениями. Однако остается проблема!

                          Релятивистская энергия – Университетская физика, Том 3

                          Цели обучения

                          К концу этого раздела вы сможете:

                          • Объясните, как теорема о работе-энергии приводит к выражению для релятивистской кинетической энергии объекта.
                          • Покажите, как релятивистская энергия соотносится с классической кинетической энергией, и устанавливает предел скорости любого объекта с массой
                          • Опишите, как полная энергия частицы связана с ее массой и скоростью
                          • Объясните, как относительность связана с эквивалентностью энергии и массы, и некоторые практические последствия эквивалентности энергии и массы.

                          Токамак на (Рисунок) представляет собой экспериментальный термоядерный реактор, который может преобразовывать массу в энергию.Ядерные реакторы – доказательство взаимосвязи энергии и материи.

                          Сохранение энергии – один из важнейших законов физики. Мало того, что энергия имеет много важных форм, каждая форма может быть преобразована в любую другую. Мы знаем, что обычно общее количество энергии в системе остается постоянным. С точки зрения релятивизма энергия по-прежнему сохраняется, но теперь необходимо учитывать эквивалентность энергии и массы, например, в реакциях, происходящих в ядерном реакторе.Релятивистская энергия намеренно определена так, чтобы она сохранялась во всех инерциальных системах отсчета, как и в случае с релятивистским импульсом. Как следствие, некоторые фундаментальные величины связаны между собой способами, не известными в классической физике. Все эти соотношения подтверждены экспериментальными результатами и имеют фундаментальные последствия. Измененное определение энергии содержит некоторые из самых фундаментальных и впечатляющих новых открытий природы в новейшей истории.

                          Национальный эксперимент со сферическим тором (NSTX) – это термоядерный реактор, в котором изотопы водорода подвергаются слиянию с образованием гелия.В этом процессе относительно небольшая масса топлива преобразуется в большое количество энергии. (Источник: Принстонская лаборатория физики плазмы)

                          Кинетическая энергия и предел скорости

                          Первый постулат относительности гласит, что законы физики одинаковы во всех инерциальных системах отсчета. Эйнштейн показал, что закон сохранения энергии частицы справедлив с точки зрения относительности, но для энергии, выраженной в терминах скорости и массы, способом, совместимым с теорией относительности.

                          Рассмотрим сначала релятивистское выражение для кинетической энергии. Мы снова используем u для скорости, чтобы отличить ее от относительной скорости v между наблюдателями. Классически кинетическая энергия связана с массой и скоростью известным выражением. Соответствующее релятивистское выражение для кинетической энергии может быть получено из теоремы работы-энергии. Эта теорема утверждает, что чистая работа системы переходит в кинетическую энергию. В частности, если сила, выраженная как ускорение частицы от состояния покоя до ее конечной скорости, работа, совершаемая над частицей, должна быть равна ее конечной кинетической энергии.В математической форме для одномерного движения:

                          Интегрируйте это по частям, чтобы получить

                          Релятивистская кинетическая энергия

                          Релятивистская кинетическая энергия любой частицы массой м равна

                          Когда объект неподвижен, его скорость равна

                          так что в покое, как положено. Но выражение для релятивистской кинетической энергии (такой как полная энергия и энергия покоя) не очень похоже на классическое.Чтобы показать, что выражение для сводится к классическому выражению для кинетической энергии при малых скоростях, мы используем биномиальное разложение, чтобы получить приближение для малых:

                          , пренебрегая очень маленькими членами и более высокими степенями выбора, и приводит к выводу, что γ на нерелятивистских скоростях, где мало, удовлетворяет

                          Биномиальное разложение – это способ выразить алгебраическую величину как сумму бесконечного ряда членов.В некоторых случаях, как здесь в пределе малой скорости, большинство членов очень маленькие. Таким образом, полученное здесь выражение для неточно, но это очень точное приближение. Следовательно, на малой скорости:

                          Если ввести это в выражение для релятивистской кинетической энергии, получим

                          То есть релятивистская кинетическая энергия становится такой же, как классическая кинетическая энергия, когда

                          Еще более интересно исследовать, что происходит с кинетической энергией, когда скорость объекта приближается к скорости света.Мы знаем, что оно становится бесконечным, когда u приближается к c , так что оно также становится бесконечным, когда скорость приближается к скорости света ((рисунок)). Увеличение намного больше, чем в v приближается к c. Бесконечное количество работы (и, следовательно, бесконечное количество подводимой энергии) требуется, чтобы разогнать массу до скорости света.

                          Скорость света

                          Ни один объект с массой не может достичь скорости света.

                          Скорость света – это предел скорости для любой частицы, имеющей массу.Все это согласуется с тем фактом, что скорости менее c всегда прибавляются к менее чем c . Как релятивистская форма кинетической энергии, так и предел скорости, равный c , были подробно подтверждены в многочисленных экспериментах. Независимо от того, сколько энергии вкладывается в ускорение массы, ее скорость может приближаться, а не достигать скорости света.

                          Этот график зависимости скорости от скорости показывает, как кинетическая энергия неограниченно увеличивается по мере приближения скорости к скорости света.Также показана классическая кинетическая энергия.

                          Сравнение кинетической энергии Электрон имеет скорость (а) Вычислите кинетическую энергию электрона в МэВ. (b) Сравните это с классическим значением кинетической энергии при этой скорости. (Масса электрона)

                          Стратегия

                          Выражение для релятивистской кинетической энергии всегда верно, но для (a) оно должно использоваться, потому что скорость очень релятивистская (близка к c ). Сначала мы вычисляем релятивистский фактор, а затем используем его для определения релятивистской кинетической энергии.Для (b) мы вычисляем классическую кинетическую энергию (которая была бы близка к релятивистскому значению, если бы v было меньше нескольких процентов от c ) и видим, что это не то же самое.

                          Решение для (а) Для части (а):

                          1. Определить известных:
                          2. Определить неизвестное:
                          3. Выразите ответ в виде уравнения: с
                          4. Сделайте расчет. Сначала вычислите. Оставьте лишние цифры, потому что это промежуточный расчет:
                            .


                            Теперь используйте это значение для расчета кинетической энергии:

                          5. Перевести единицы:

                          Решение для (b) Для части (b):

                          1. Перечислить известных:
                          2. Список неизвестного:
                          3. Выразите ответ в виде уравнения:
                          4. Сделайте расчет:
                          5. Перевести единицы:

                          Значение Как и следовало ожидать, потому что скорость равна 99.0% скорости света, классическая кинетическая энергия значительно отличается от правильного релятивистского значения. Отметим также, что классическое значение намного меньше релятивистского. Собственно, в данном случае. Это показывает, насколько сложно заставить массу двигаться со скоростью, близкой к скорости света. Требуется гораздо больше энергии, чем предсказывалось классически. Чтобы приблизить скорость массы к скорости света, требуется постоянно увеличивающееся количество энергии. Энергия в 3 МэВ – очень небольшая величина для электрона, и ее можно достичь с помощью современных ускорителей частиц.SLAC, например, может ускорять электроны до более чем

                          Есть ли смысл приближать v к c , чем на 99,0% или 99,9%? Ответ положительный. Делая это, мы многому научимся. Энергия, которая переходит в высокоскоростную массу, может быть преобразована в любую другую форму, в том числе в совершенно новые частицы. В Большом адронном коллайдере (рисунок) заряженные частицы ускоряются до того, как войдут в кольцеобразную структуру. Там два пучка частиц ускоряются до своей конечной скорости около 99.7% скорости света в противоположных направлениях, и они сталкиваются, производя совершенно новые виды частиц. Таким образом мы узнали большую часть того, что мы знаем о субструктуре материи и о сборе экзотических короткоживущих частиц в природе. Паттерны в характеристиках этих ранее неизвестных частиц намекают на основную субструктуру всей материи. Эти частицы и некоторые их характеристики будут обсуждаться в следующей главе, посвященной физике элементарных частиц.

                          Европейская организация ядерных исследований (названная ЦЕРН по французскому названию) управляет крупнейшим в мире ускорителем элементарных частиц, расположенным на границе между Францией и Швейцарией.(кредит: модификация работы НАСА)

                          Полная релятивистская энергия

                          Выражение для кинетической энергии можно изменить на:

                          Эйнштейн утверждал в отдельной статье, также позже опубликованной в 1905 году, что если энергия частицы изменяется в зависимости от ее массы, то многочисленные экспериментальные данные с тех пор подтверждают, что это соответствует энергии, которую имеет частица с массой m в состоянии покоя. . Например, когда нейтральный пион с массой m в состоянии покоя распадается на два фотона, фотоны имеют нулевую массу, но наблюдается полная энергия, соответствующая пиону.Точно так же, когда частица массой m распадается на две или более частицы с меньшей общей массой, наблюдаемая кинетическая энергия, сообщаемая продуктам распада, соответствует уменьшению массы. Таким образом, E – это полная релятивистская энергия частицы, а это энергия ее покоя.

                          Общая энергия

                          Полная энергия E частицы

                          , где m – масса, c – скорость света и u – скорость массы относительно наблюдателя.

                          Энергия отдыха

                          Энергия покоя объекта

                          Это правильная форма самого известного уравнения Эйнштейна, которое впервые показало, что энергия связана с массой покоящегося объекта. Например, если в объекте накапливается энергия, его масса покоя увеличивается. Это также означает, что масса может быть разрушена для высвобождения энергии. Значение этих первых двух уравнений относительно релятивистской энергии настолько велико, что они не были полностью признаны в течение нескольких лет после того, как Эйнштейн опубликовал их в 1905 году, равно как и экспериментальное доказательство их правильности поначалу не было широко признано.Следует отметить, что Эйнштейн действительно понял и описал смысл и значение своей теории.

                          Сегодня практическое применение для преобразования массы в другую форму энергии , например, в ядерном оружии и атомных электростанциях, хорошо известно. Но были и примеры, когда Эйнштейн впервые предложил правильную форму релятивистской энергии, и он описал некоторые из них. Ядерное излучение было открыто в предыдущее десятилетие, и оставалось загадкой, откуда берется его энергия.Объяснение состояло в том, что в некоторых ядерных процессах небольшое количество массы разрушается, а энергия высвобождается и переносится ядерным излучением. Но количество уничтоженной массы настолько мало, что трудно обнаружить, что что-то отсутствует. Хотя Эйнштейн предположил, что это источник энергии в радиоактивных солях, которые тогда изучались, прошло много лет, прежде чем было широкое признание того, что масса может быть и, фактически, обычно преобразуется в энергию ((Рисунок)).

                          (а) Солнце и (б) Паровая электрическая станция Саскуэханна преобразуют массу в энергию – Солнце посредством ядерного синтеза, а электрическая станция – посредством ядерного деления.(кредит а: модификация работы NASA / SDO (AIA))

                          Из-за отношения энергии покоя к массе мы теперь считаем массу формой энергии, а не чем-то отдельным. До работы Эйнштейна на это не было даже намека. Эквивалентность энергии и массы, как теперь известно, является источником солнечной энергии, энергии ядерного распада и даже одним из источников энергии, поддерживающих жар внутри Земли.

                          Накопленная энергия и потенциальная энергия

                          Что происходит с энергией, хранящейся в неподвижном объекте, например, с энергией, передаваемой в батарею при ее зарядке, или с энергией, хранящейся в сжатой пружине игрушечного пистолета? Подводимая энергия становится частью общей энергии объекта и, таким образом, увеличивает его массу покоя.Вся накопленная и потенциальная энергия становится массой в системе. В кажущемся противоречии принцип сохранения массы (то есть общая масса постоянна) был одним из великих законов, проверенных наукой девятнадцатого века. Почему не было замечено, что это неверно? Следующий пример помогает ответить на этот вопрос.

                          Расчет массы покоя Автомобильный аккумулятор рассчитан на способность перемещать 600 ампер-часов заряда при напряжении 12,0 В. (a) Рассчитать увеличение массы покоя такой батареи, когда она берется от полностью разряженной до полностью заряженной. при условии, что ни один из химических реагентов не попадет в аккумулятор и не покинет его.(б) Какой это процент увеличения при массе батареи 20,0 кг?

                          Стратегия

                          В части (а) мы сначала должны найти энергию, запасенную в виде химической энергии в батарее, которая равна электрической энергии, которую может обеспечить батарея. Потому что мы должны рассчитать заряд q , в котором произведение текущего I и времени t . Затем мы умножаем результат на 12,0 В. Затем мы можем рассчитать увеличение массы батареи, используя Часть (b) – это простое соотношение, преобразованное в процент.

                          Решение для (а)

                          1. Определить известных:
                          2. Определить неизвестное:
                          3. Выразите ответ в виде уравнения:
                          4. Сделайте расчет:


                            Запишите амперы A в кулонах в секунду (Кл / с) и преобразуйте часы в секунды:


                            , где мы использовали преобразование

                          Решение для (b) Для части (b):

                          1. Определить известных:
                          2. Определить неизвестное:% изменение.
                          3. Выразите ответ в виде уравнения:
                          4. Сделайте расчет:

                          Значение И фактическое увеличение массы, и процент увеличения очень малы, потому что энергия делится на очень большое число. Мы должны иметь возможность измерить массу батареи с точностью до миллиардной доли процента или одной части, чтобы заметить это увеличение. Неудивительно, что изменение массы не наблюдается. Фактически, это изменение массы настолько мало, что мы можем задаться вопросом, как можно проверить его реальность.Ответ находится в ядерных процессах, в которых процент разрушенной массы достаточно велик, чтобы его можно было точно измерить. Например, масса топлива ядерного реактора заметно меньше, когда используется его энергия. В этом случае накопленная энергия была высвобождена (преобразована в основном в тепловую энергию для питания электрогенераторов), а масса покоя уменьшилась. Уменьшение массы также происходит за счет использования энергии, накопленной в батарее, за исключением того, что накопленная энергия намного больше в ядерных процессах, что делает изменение массы измеримым как на практике, так и в теории.

                          Релятивистская энергия и импульс

                          Классически мы знаем, что кинетическая энергия и импульс связаны друг с другом, потому что:

                          С точки зрения релятивизма, мы можем получить связь между энергией и импульсом, алгебраически манипулируя их определяющими уравнениями. Это дает:

                          , где E – релятивистская полная энергия, а p – релятивистский импульс. Эта связь между релятивистской энергией и релятивистским импульсом более сложна, чем классическая версия, но мы можем получить некоторые интересные новые идеи, исследуя ее.Во-первых, полная энергия связана с импульсом и массой покоя. В состоянии покоя импульс равен нулю, и уравнение дает полную энергию как энергию покоя (так что это уравнение согласуется с обсуждением энергии покоя выше). Однако по мере того, как масса ускоряется, ее импульс p увеличивается, таким образом увеличивая полную энергию. При достаточно высоких скоростях член энергии покоя становится пренебрежимо малым по сравнению с членом импульса, таким образом, при чрезвычайно релятивистских скоростях.

                          Если мы считаем, что импульс p отличается от массы, мы можем определить значение уравнения для частицы, не имеющей массы.Если мы возьмем m равным нулю в этом уравнении, то безмассовые частицы будут иметь этот импульс. В природе существует несколько безмассовых частиц, включая фотоны (которые представляют собой пакеты электромагнитного излучения). Другое значение состоит в том, что безмассовая частица должна двигаться со скоростью c и только со скоростью c . Подробное изучение взаимосвязи в уравнении выходит за рамки этого текста, но вы можете видеть, что эта взаимосвязь имеет важные последствия для специальной теории относительности.

                          Проверьте свое понимание Какова кинетическая энергия электрона, если его скорость равна 0,992 c ?

                          Концептуальные вопросы

                          Как классические законы сохранения энергии и массы модифицируются современной теорией относительности?

                          Что происходит с массой воды в кастрюле, когда она остывает, если молекулы не выходят или не добавляются? Наблюдается ли это на практике? Объяснять.

                          Поскольку он теряет тепловую энергию, которая является кинетической энергией случайного движения составляющих его частиц, его масса уменьшается на чрезвычайно малую величину, что описывается эквивалентностью энергии и массы.

                          Рассмотрим мысленный эксперимент. Ранним утром вы кладете расширенный воздушный шар на весы на улице. Воздушный шар остается на весах, и вы можете измерить изменение его массы. Изменится ли масса воздушного шара в течение дня? Обсудите трудности в проведении этого эксперимента.

                          Масса топлива в ядерном реакторе уменьшается на заметную величину по мере выделения энергии. Верно ли то же самое для угля и кислорода, объединенного на обычной электростанции? Если да, то наблюдается ли это на практике для угля и кислорода? Объяснять.

                          Да, в принципе, любое уменьшение энергии будет иметь аналогичный эффект на массу, но это изменение будет настолько незначительным для изменений энергии в химической реакции, что на практике оно не будет обнаружено.

                          Мы знаем, что скорость объекта с массой имеет верхний предел c . Есть ли верхний предел его импульса? Его энергия? Объяснять.

                          Учитывая тот факт, что свет движется со скоростью c , может ли он иметь массу? Объяснять.

                          Не согласно специальной теории относительности.Ничто с массой не может достичь скорости света.

                          Если вы используете наземный телескоп для проецирования лазерного луча на Луну, вы можете перемещать пятно по поверхности Луны со скоростью, превышающей скорость света. Нарушает ли это современную теорию относительности? (Обратите внимание, что свет посылается с Земли на Луну, а не через поверхность Луны.)

                          Проблемы

                          Какова энергия покоя электрона с учетом его массы. Ответьте в джоулях и МэВ.

                          0,512 МэВ по указанному количеству значащих цифр. Точное значение ближе к 0,511 МэВ.

                          Найдите энергию покоя протона в джоулях и МэВ, учитывая его массу

                          .

                          Если энергии покоя протона и нейтрона (двух составляющих ядра) равны 938,3 и 939,6 МэВ соответственно, какова разница в их массе в килограммах?

                          до двух цифр, потому что разница в энергии массы покоя определяется до двух цифр

                          По оценкам, Большой взрыв, положивший начало Вселенной, вызвал высвобождение энергии.Сколько звезд могла бы создать половина этой энергии, если предположить, что средняя масса звезды равна?

                          (a) Используя данные Потенциальной энергии системы, вычислите массу, преобразованную в энергию при делении 1,00 кг урана. (b) Каково отношение уничтоженной массы к исходной массе,

                          Приблизительно количество энергии, получаемой в результате синтеза водорода в Мировом океане. а) Если бы эта энергия была использована, как бы уменьшилась масса океанов? б) Какому количеству воды это соответствует? (c) Прокомментируйте, составляет ли это значительную часть общей массы Мирового океана.

                          Мюон имеет энергию покоя 105,7 МэВ, и он распадается на электрон и безмассовую частицу. (а) Если вся потерянная масса конвертируется в кинетическую энергию электрона, найдите электрон. б) Какова скорость электрона?

                          а. 208; б. 0,999988 c ; шесть цифр используются для отображения отличия от c

                          A -мезон – частица, распадающаяся на мюон и безмассовую частицу. -Мезон имеет энергию массы покоя 139,6 МэВ, а мюон имеет энергию массы покоя 105.7 МэВ. Предположим, что -мезон находится в состоянии покоя, и вся недостающая масса уходит в кинетическую энергию мюона. Как быстро будет двигаться мюон?

                          (a) Вычислите релятивистскую кинетическую энергию 1000-кг автомобиля, движущегося со скоростью 30,0 м / с, если бы скорость света была всего 45,0 м / с. (б) Найдите отношение релятивистской кинетической энергии к классической.

                          а. б. 1,54

                          Альфа-распад – это ядерный распад, при котором испускается ядро ​​гелия. Если ядро ​​гелия имеет массу и дается 5.00 МэВ кинетической энергии, какова его скорость?

                          (a) Бета-распад – это ядерный распад, при котором испускается электрон. Если электрону дать 0,750 МэВ кинетической энергии, какова его скорость? (b) Прокомментируйте, как высокая скорость согласуется с кинетической энергией по сравнению с энергией массы покоя электрона.

                          а. 0,914 c ; б. Энергия массы покоя электрона составляет 0,511 МэВ, поэтому кинетическая энергия составляет приблизительно 150% энергии массы покоя. Электрон должен двигаться со скоростью, близкой к скорости света.

                          Дополнительные проблемы

                          (a) Какая относительная скорость (b) Какая относительная скорость равна

                          (a) Какая относительная скорость (б) Какая относительная скорость равна

                          Необоснованные результаты (a) Найдите значение, требуемое для следующей ситуации. Наблюдатель с Земли измеряет, что прошло 23,9 часа, в то время как сигналы от высокоскоростного космического зонда показывают, что на борту прошло 24,0 часа. б) Что неразумного в этом результате? (c) Какие предположения необоснованны или непоследовательны?

                          (a) Сколько времени нужно космонавту на (Рис.), Чтобы путешествовать 4.30 св. Лет (по измерениям наземного наблюдателя)? б) Сколько времени это займет, по мнению космонавта? (c) Убедитесь, что эти два времени связаны через замедление времени с заданным.

                          а. 4.303 y до четырех цифр, чтобы показать любой эффект; б. 0,1434 л; c.

                          (a) Насколько быстро спортсмену нужно бегать на 100-километровую дистанцию, чтобы выглядеть на 100 ярдов в длину? б) Соответствует ли ответ тому факту, что релятивистские эффекты трудно наблюдать в обычных обстоятельствах? Объяснять.

                          (a) Найдите значение для следующей ситуации.Космонавт измеряет длину своего космического корабля в 100 м, а наземный наблюдатель – в 25,0 м. б) Какова скорость космического корабля относительно Земли?

                          а. 4.00; б.

                          Часы космического корабля идут в десять раз быстрее, чем те же часы на Земле. Какая скорость у космического корабля?

                          Частота сердечных сокращений астронавта составляет 66 ударов в минуту, что было измерено во время его физического осмотра на Земле. Частота сердцебиения космонавта измеряется, когда он находится на космическом корабле, летящем на нулевой отметке.5 c относительно Земли наблюдателем (A) на корабле и наблюдателем (B) на Земле. (a) Опишите экспериментальный метод, с помощью которого наблюдатель B на Земле сможет определить частоту сердцебиения космонавта, когда космонавт находится на космическом корабле. (b) Какую частоту сердечных сокращений космонавта сообщат наблюдатели A и B?

                          а. A посылает радиоимпульс при каждом сердечном сокращении к B, который знает их относительную скорость и использует формулу замедления времени для вычисления надлежащего временного интервала между сердечными сокращениями по наблюдаемому сигналу.б.

                          Космический корабль (A) движется со скоростью c / 2 по отношению к другому космическому кораблю (B). Наблюдатели в A и B устанавливают свои часы так, чтобы событие в ( x, y, z, t ) включения лазера на космическом корабле B имело координаты (0 , 0 , 0 , 0) в A, а также (0 , 0 , 0 , 0) в B. Наблюдатель в начале координат B включает лазер и выключает его в свое время. Какова продолжительность времени между включением и выключением с точки зрения наблюдателя в точке A?

                          Те же два наблюдателя, что и в предыдущих упражнениях.Стержень длиной 1 м размещен по оси x в кадре B от начала координат до Какова длина стержня, наблюдаемого наблюдателем в кадре космического корабля A?

                          Наблюдатель видит два события на расстоянии 800 м друг от друга. Насколько быстро должен двигаться второй наблюдатель относительно первого, чтобы увидеть, как два события происходят одновременно?

                          Наблюдатель, стоящий у железнодорожных путей, видит, как две молнии одновременно ударяют по концам поезда длиной 500 м в тот момент, когда его середина проезжает со скоростью 50 м / с.Используйте преобразование Лоренца, чтобы найти время между ударами молнии, измеренное пассажиром, сидящим в середине поезда.


                          , поскольку мы можем проигнорировать этот термин и найти

                          Нарушение ньютоновской одновременности пренебрежимо мало, но не совсем равно нулю при реалистичной скорости поезда 50 м / с.

                          Два астрономических события наблюдаются с Земли, которые происходят с интервалом в 1 с и на расстоянии друг от друга. (а) Определите, является ли разделение двух событий пространственным или временным.(b) Укажите, что это означает в отношении того, согласуется ли со специальной теорией относительности, что одно событие вызвало другое?

                          Два астрономических события наблюдаются с Земли, которые происходят с интервалом 0,30 с и на расстоянии друг от друга. Насколько быстро космический корабль должен перемещаться от места одного события к другому, чтобы события происходили одновременно, при измерении в системе отсчета космического корабля?

                          Космический аппарат начинает движение из состояния покоя в начале координат и ускоряется с постоянной скоростью g. , если смотреть с Земли, принимается за инерциальную систему отсчета, пока не достигнет скорости c / 2.(а) Покажите, что приращение собственного времени связано с затраченным временем в системе отсчета Земли следующим образом:

                          (b) Найдите выражение для времени, затрачиваемого на достижение скорости c / 2, как это видно в системе координат Земли. (c) Используйте соотношение в (a), чтобы получить аналогичное выражение для истекшего собственного времени для достижения c /2, как видно на космическом корабле, и определите отношение времени, наблюдаемого с Земли, и времени на космическом корабле для достижения конечная скорость.

                          (a) Все галактики, кроме ближайших, удаляются от нашей Галактики Млечный Путь.Если далекая галактика удаляется от нас на 0,900 c , с какой скоростью относительно нас мы должны послать исследовательский зонд, чтобы приблизиться к другой галактике на 0,990 c , если отсчитывать от этой галактики? (б) Сколько времени потребуется зонду, чтобы достичь другой галактики, если считать с Земли? Вы можете предположить, что скорость другой галактики остается постоянной. (c) Сколько времени потребуется, чтобы радиосигнал вернулся? (Все это в принципе возможно, но не на практике.)

                          Обратите внимание, что все ответы на эту проблему представлены в виде пяти значащих цифр, чтобы различать результаты.а. 0,99947 c ; б. c.

                          Предположим, что космический корабль, направляющийся прямо к Земле на 0,750 c , может выстрелить из канистры на 0,500 c относительно корабля. (а) Какова скорость канистры относительно Земли, если она стреляет прямо в Землю? (б) Если он снят прямо с Земли?

                          Повторите предыдущую задачу с кораблем, идущим прямо от Земли.

                          а. –0,400 c ; б. –0,909 c

                          Если космический корабль приближается к Земле на 0.100 c и капсула сообщения отправляется к нему на 0,100 c относительно Земли, какова скорость капсулы относительно корабля?

                          (а) Предположим, что скорость света составляет всего 3000 м / с. Реактивный истребитель, движущийся к цели на земле со скоростью 800 м / с, стреляет пулями, каждая из которых имеет начальную скорость 1000 м / с. Какая скорость пули относительно цели? б) Если бы скорость света была такой маленькой, наблюдали бы вы релятивистские эффекты в повседневной жизни? Обсуждать.

                          а. 1,65 км / с; б. Да, если бы скорость света была такой маленькой, скорости, которых мы можем достичь в повседневной жизни, были бы больше 1% от скорости света, и мы могли бы наблюдать релятивистские эффекты гораздо чаще.

                          Если галактика, удаляющаяся от Земли, имеет скорость 1000 км / с и излучает свет с длиной волны 656 нм, характерный для водорода (наиболее распространенного элемента во Вселенной). а) Какую длину волны мы наблюдали бы на Земле? б) Что это за электромагнитное излучение? (c) Почему скорость Земли на ее орбите здесь незначительна?

                          Космический зонд, движущийся к ближайшей звезде, движется и передает радиоинформацию на частоте вещания 1.00 ГГц. Какая частота принимается на Земле?

                          Рядом с центром нашей галактики газообразный водород движется прямо от нас по своей орбите вокруг черной дыры. Мы получаем электромагнитное излучение 1900 нм и знаем, что оно было 1875 нм, когда испускалось газообразным водородом. Какая скорость газа?

                          (a) Вычислите скорость частицы пыли, которая имеет тот же импульс, что и протон, движущийся с координатой 0,999 c . б) Что малая скорость говорит нам о массе протона по сравнению даже с крошечным количеством макроскопической материи?

                          а.б. Малая скорость говорит нам о том, что масса белка существенно меньше, чем масса даже крошечного количества макроскопического вещества.

                          (a) Вычислите для протона с импульсом, равным (b) Какова его скорость? Такие протоны образуют редкий компонент космического излучения с неопределенным происхождением.

                          Покажите, что релятивистская форма второго закона Ньютона: (a) (b) Найдите силу, необходимую для ускорения массы 1 кг на 1, когда она движется со скоростью c /2.

                          а.

                          б.

                          Позитрон – это антивещественная версия электрона, имеющая точно такую ​​же массу. Когда позитрон и электрон встречаются, они аннигилируют, превращая всю свою массу в энергию. (а) Найдите выделившуюся энергию, считая, что кинетическая энергия до аннигиляции пренебрежимо мала. б) Если эта энергия передана протону в виде кинетической энергии, какова его скорость? (c) Если эта энергия передана другому электрону в виде кинетической энергии, какова его скорость?

                          Какова кинетическая энергия в МэВ живущего π-мезона, измеренного в лаборатории, и когда он находится в состоянии покоя относительно наблюдателя, учитывая, что его энергия покоя составляет 135 МэВ?

                          Найдите кинетическую энергию в МэВ нейтрона с измеренной продолжительностью жизни 2065 с, учитывая, что его энергия покоя равна 939.6 МэВ, а остальная продолжительность жизни – 900 с.

                          Один нейтрон космических лучей имеет скорость относительно Земли. а) Какова полная энергия нейтрона в МэВ? (б) Найдите его импульс. (c) Находится в этой ситуации? Обсудите в терминах уравнения, приведенного в части (а) предыдущей задачи.

                          Что означает протон с энергией массы 938,3 МэВ, ускоренный за счет эффективного потенциала 1,0 TV (теравольт)?

                          (а) Каков эффективный ускоряющий потенциал электронов на Стэнфордском линейном ускорителе, если для них? (б) Какова их полная энергия (в данном случае почти такая же, как кинетическая) в ГэВ?

                          (a) Используя данные из «Потенциальной энергии системы», найдите массу, уничтоженную при высвобождении энергии в барреле сырой нефти.(b) Учитывая, что эти бочки содержат 200 литров, а плотность сырой нефти равна отношению разрушенной массы к исходной массе,

                          (a) Рассчитайте энергию, выделяемую при разрушении 1,00 кг массы. б) Сколько килограммов можно поднять на высоту 10 км с помощью этого количества энергии?

                          Ускоритель Ван де Граафа использует разность потенциалов 50,0 МВ для ускорения заряженных частиц, таких как протоны. а) Какова скорость протона, ускоряемого таким потенциалом? б) Электрон?

                          а.0,314 c ; б. 0,99995 c (пять цифр используются для отображения отличия от c )

                          Предположим, вы потребляете в своем доме в среднем электроэнергию в месяц. (а) Как долго вам хватит 1,00 г массы, преобразованной в электрическую энергию с КПД 38,0%? (б) Сколько домов можно было бы снабжать по месячной ставке в течение одного года за счет энергии, полученной в результате описанного преобразования массы?

                          (a) Атомная электростанция преобразует энергию ядерного деления в электричество с КПД 35.0%. Сколько массы разрушается за один год для непрерывного производства 1000 МВт электроэнергии? (b) Как вы думаете, можно ли было бы наблюдать эту потерю массы, если бы общая масса топлива составляла

                          ?

                          а. 1,00 кг; б. Эту большую массу можно было бы измерить, но, вероятно, ее нельзя было бы наблюдать, просто глядя, потому что она составляет 0,01% от общей массы.

                          Ракеты с ядерными двигателями исследовались в течение нескольких лет, прежде чем вопросы безопасности стали первостепенными. а) Какая часть массы ракеты должна быть уничтожена, чтобы вывести ее на низкую околоземную орбиту, если не учитывать уменьшение силы тяжести? (Предположим, что высота орбиты составляет 250 км, и рассчитайте как кинетическую энергию (классическую), так и необходимую гравитационную потенциальную энергию.) (b) Если корабль имеет массу (100 тонн), какой ядерный взрыв общей мощности в тоннах тротила необходим?

                          Солнце вырабатывает энергию в размере Вт за счет синтеза водорода. Около 0,7% каждого килограмма водорода переходит в энергию, вырабатываемую Солнцем. а) Сколько килограммов водорода плавится каждую секунду? (б) Если Солнце на 90,0% состоит из водорода, и половина из него может подвергнуться термоядерному превращению до того, как солнце изменит свой характер, как долго оно сможет производить энергию с нынешней скоростью? (c) Сколько килограммов массы теряет Солнце за секунду? (d) Какую часть своей массы он потеряет за время, указанное в части (b)?

                          Покажите, что для частицы инвариантен относительно преобразований Лоренца.

                          Глоссарий

                          релятивистская кинетическая энергия
                          кинетическая энергия объекта, движущегося с релятивистскими скоростями
                          энергия покоя
                          энергии, запасенной в покоящемся объекте:
                          скорость света
                          Предел максимальной скорости для любой частицы с массой
                          общая энергия
                          сумма всех энергий частицы, включая энергию покоя и кинетическую энергию, заданных для частицы с массой m и скоростью u где
                          .

                          Оставить комментарий